Exam 3 Ch. 10, 14, 16 (evolve chapters 14, 15, 28, 31, 32, 33, 37)

अब Quizwiz के साथ अपने होमवर्क और परीक्षाओं को एस करें!

Which statements would not be advisable to use as a basis for therapeutic discussion following a perinatal loss? (Select all that apply.) A. "This must be hard for you" B. "I'm sorry" C. "You're young, you can have other children" D. "You wanted a boy anyway, so now you have another chance" E. "I am sad for you"

"This must be hard for you," "I'm sorry," and "I am sad for you" are acceptable statements following perinatal loss. "You're young, you can have other children" and "You wanted a boy anyway, so now you have another chance" would not be considered therapeutic. C, D

A woman presents to the emergency department complaining of bleeding and cramping. The initial nursing history is significant for a last menstrual period 6 weeks ago. On sterile speculum examination, the primary health care provider finds that the cervix is closed. The anticipated plan of care for this woman is based on a probable diagnosis of which type of spontaneous abortion? A. Incomplete B. Inevitable C. Threatened D. Septic

A woman with a threatened abortion presents with spotting, mild cramps, and no cervical dilation. A woman with an incomplete abortion presents with heavy bleeding, mild to severe cramping, and cervical dilation. An inevitable abortion presents with the same symptomatology as an incomplete abortion: heavy bleeding, mild to severe cramping, and cervical dilation. A woman with a septic abortion presents with malodorous bleeding and typically a dilated cervix. C

A nurse is assessing a pregnant woman during a prenatal visit. Several presumptive indicators of pregnancy are documented. Presumptive indicators include: (Circle all that apply) A. Nausea and vomiting B. Quickening C. Hegar sign D. Palpation of fetal movement by the nurse E. Amenorrhea F. Positive pregnancy test

A, B, E

What is the expected delivery date for a pregnant woman whose first day of her last menstrual period was 20th April, 2014? A. 27th December, 2014 B. 20th January, 2015 C. 27th January, 2015 D. 7th February, 2016

According to Nagele's rule, the expected date of delivery is calculated by adding 7 days and 9 months to the first day of the last menstrual period. Because the patient's last menstrual period was 20th April, 2014, the expected date of delivery would be 20th April, 2014 + 9 months + 7 days = 27th January, 2015. C

The perinatal nurse knows that the blood volume in pregnancy increases on average by: A. 20% to 30% B. 30% to 40% C. 40% to 50% D. 50% to 60%

C

The nurse is assessing a client with couvade syndrome. What symptoms is the nurse likely to find? Select all that apply. A. Nausea B. Skin rashes C. Sore throat D. Weight gain E. Persistent cough

Couvade syndrome is a condition in which men experience pregnancy-like symptoms, such as nausea, weight gain, and other physical symptoms. During this condition some emotional and physiologic changes are observed in men. Couvade syndrome does not have any impact on the skin or throat. Therefore, the client will not have skin rashes, sore throat, or persistent cough. A, D

Which ratio would be used to restore effective circulating volume in a postpartum client who is experiencing hypovolemic shock? A. 4:1 B. 2:1 C. 1:1 D. 3:1

D

The nurse is caring for a pregnant client receiving anticoagulant therapy. On reviewing the client's lab reports, the nurse finds an abrupt increase in clotting time. What does the nurse suspect that might be the reason for this? A. Consumption of eggs B. Consumption of meat C. Consumption of ginger D. Consumption of caffeine

Ginger has anticoagulant properties. The client who is receiving anticoagulant therapy and consuming ginger may experience a synergistic effect that leads to a prolonged clotting time. Therefore, the client should not consume ginger. The consumption of eggs and meat helps provide nutrition to the client. The consumption of caffeine does not inhibit coagulation. However, a breastfed infant may develop anemia if the mother consumes caffeine. C

A woman diagnosed with marginal placenta previa gave birth vaginally 15 minutes ago. At present she is at the greatest risk for: A. Hemorrhage. B. Infection. C. Urinary retention. D. Thrombophlebitis.

Hemorrhage is the most immediate risk because the lower uterine segment has limited ability to contract to reduce blood loss. Infection is a risk because of the location of the placental attachment site; however, it is not a priority concern at this time. Placenta previa poses no greater risk for urinary retention or thrombophlebitis than does a normally implanted placenta. A

The most prevalent clinical manifestation of abruptio placentae (as opposed to placenta previa) is: A. Bleeding. B. Intense abdominal pain. C. Uterine activity. D. Cramping.

Pain is absent with placenta previa and may be agonizing with abruptio placentae. Bleeding, uterine activity, and cramping may be present in varying degrees for both placental conditions. B

While assisting the primary health care provider, the nurse documents the subjective symptoms of pregnancy in a client. Which subjective symptom does the nurse record based on the client's statement? A. Vaginal changes B. Urinary frequency C. Breast enlargement D. Abdominal enlargement

Subjective symptoms are symptoms that can be reported by the client. Subjective symptoms of pregnancy include urinary frequency, nausea and vomiting, and fatigue. Vaginal changes, breast enlargement, and abdominal enlargement are objective signs, which can be observed. B

A 28-year-old multipara delivered a 9 pound, 3 ounce baby girl an hour ago after a 22-hour labor with a forceps-assisted birth. As the patient is holding her daughter, she keeps shifting position and is becoming increasingly irritable and annoyed with everyone in the room. What action should the nurse initially take? A. Massage the fundus B. Check her perineum. C. Assess her vital signs. D. Check the tone of her fundus.

The client is exhibiting increasing anxiety, which can signal the presence of postpartum hemorrhage. Risk factors for postpartum hemorrhage include a large fetus, prolonged labor, and a forceps-assisted birth. Because vital signs change late, the fastest way to see the amount of current hemorrhage is to check the perineum. The fundus would be massaged and additional nursing and medical interventions would be instituted. B

What dietary instructions does the nurse give a client who has experienced miscarriage? A. "Increase folic acid and zinc in your diet." B. "Eat foods that are high in iron and protein." C. "Restrict fluid intake to 125 milliliters per hour." D. "Reduce dietary fat intake by 40 to 50 grams per day."

The nurse advises the client to eat foods that are high in iron and protein to help in tissue repair and red blood cell (RBC) replacement. Folic acid and zinc are more helpful in maintaining a pregnancy and are not needed after a miscarriage. Fluid intake is restricted in clients who are at risk for pulmonary edema. Dietary fat is reduced if the client has cholecystitis or cholelithiasis, because it may worsen epigastric pain. B

A woman gave birth to twin girls, one of whom was stillborn. Which nursing action would be least helpful in supporting the woman as she copes with her loss? A. Remind her that she should be happy that one daughter survived and is healthy. B. Assist the woman to take pictures of both babies C. Encourage the woman to hold the deceased twin in her arms to say goodbye D. Offer her the opportunity for counseling to help her grief and the grief of the surviving twin as she gets older.

A

A woman in her first trimester of pregnancy complains of nausea. Which suggestion by the nurse is best? A. "Try eating plain, dry crackers before you get out of bed". B. "You may be constipated; try some senna tablets". C. "I heard that licorice root is good for nausea". D. "Don't worry unless you start vomiting continuously".

A

A woman's labor is being induced. The nurse assesses the woman's status and that of her fetus and the labor process just before an infusion increment of 2 milliunits/minute. The nurse discontinues the infusion and notifies the woman's primary health provider if during this assessment she notes: A. Frequency of uterine contractions: every 1 1/2 minutes B. Variability of fetal heart rate (FHR): present C. Deceleration patterns: early decelerations noted with several contractions D. Intensity of uterine contractions at their peaks: 80 to 85 mm hg

A

A thrombosis results from the formation of a blood clot or clots inside a blood vessel and is caused by inflammation or partial obstruction of the vessel. Three thromboembolic conditions are of concern during the postpartum period; which of the following is not? A. Amniotic fluid embolism (AFE) B. Superficial venous thrombosis C. Deep vein thrombosis D. Pulmonary embolism

An AFE occurs during the intrapartum period, when amniotic fluid containing particles of debris enters the maternal circulation. Although AFE is rare, the mortality rate is as high as 80%. A superficial venous thrombosis includes involvement of the superficial saphenous venous system. With deep vein thrombosis, the involvement varies but can extend from the foot to the iliofemoral region. A pulmonary embolism is a complication of deep vein thrombosis, occurring when part of a blood clot dislodges and is carried to the pulmonary artery, where it occludes the vessel and obstructs blood flow to the lungs. A

A client who is 32 weeks pregnant visits a maternal clinic for a routine health checkup. The ultrasound and magnetic resonance imagining (MRI) reveal that the woman is at risk of placenta accreta. Which intervention should be performed during the delivery to ensure client's safety? A. Blood transfusion B. Hysterectomy after delivery C. Natural removal of the placenta D. Administration of uterine contractile drugs

Artificial removal of the placenta leads to excessive bleeding, which can result in hemorrhage and sometimes death. So, the primary health care provider teaches the surgical team about the risk of placenta accrete and instructs them to remove the placenta naturally to reduce bleeding. Blood transfusion is a common intervention during labor that is used to restore blood volume and components. A hysterectomy is done when the bleeding is uncontrollable. Uterine contractile drugs, such asoxytocin (Pitocin), are administered to induce labor, and are not specific to clients who are at risk of placenta accrete. C

A 17-year-old woman experiences a miscarriage at 12 weeks of gestation. When she is informed about the miscarriage she begins to cry, stating that she was upset about her pregnancy at first and now is being punished for not wanting her baby. The nurse's best response is: A. "You are still young, you probably were not ready for a baby right now" B. "This must be so hard for you - I am here if you want to talk" C. "At least this happened early in your pregnancy before you felt your baby move". D. "God must have a good reason for letting this happen"

B

A woman at 26 weeks of gestation is being assessed to determine whether she is experiencing preterm labor. What finding indicates that preterm labor is occurring? A. Estriol is not found in maternal saliva. B. Irregular, mild uterine contractions are occurring every 12 to 15 minutes. C. Fetal fibronectin is present in vaginal secretions . D. The cervix is effaced 50% and dilated to 3 cm.

Cervical changes such as shortened endocervical length, effacement, and dilation are predictors of imminent preterm labor. Changes in the cervix accompanied by regular contractions indicate labor at any gestation. Irregular, mild contractions that do not cause cervical change are not considered a threat. Estriol is a form of estrogen produced by the fetus that is present in plasma at 9 weeks of gestation. Levels of salivary estriol have been shown to increase before preterm birth. The presence of fetal fibronectin in vaginal secretions between 24 and 36 weeks of gestation could predict preterm labor, but it has only a 20% to 40% positive predictive value. Of more importance are other physiologic clues of preterm labor, such as cervical changes. D

When counseling a client about getting enough iron in her diet, the maternity nurse should tell her that: A. Milk, coffee, and tea aid iron absorption if consumed at the same time as iron. B. Iron absorption is inhibited by a diet rich in vitamin C. C. Iron supplements are permissible for children in small doses. D. Constipation is common with iron supplements.

Constipation can be a problem with iron supplements. Milk, coffee, and tea actually inhibit iron absorption when consumed at the same time as iron. Vitamin C promotes iron absorption. Children who ingest iron can get very sick and even die. D

What is the treatment that should be considered first for the woman with von Willebrand disease who experiences a postpartum hemorrhage? A. Cryoprecipitate B. Factor VIII and vWf C. Desmopressin D. Hemabate

Cryoprecipitate may be used; however, because of the risk of possible donor viruses, other modalities are considered safer. Treatment with plasma products, such as factor VIII and vWf, is an acceptable option for this client. Because of the repeated exposure to donor blood products and possible viruses, this is not the initial treatment of choice. Desmopressin is the primary treatment of choice. This hormone can be administered orally, nasally, and intravenously. This medication promotes the release of factor VIII and vWf from storage. Although the administration of this prostaglandin is known to promote contraction of the uterus during postpartum hemorrhage, it is not effective for the client who presents with a bleeding disorder. C

A pregnant client is on tocolytic therapy with magnesium sulfate. Under which client circumstance would the nurse suggest discontinuing the therapy? A. Blood pressure is 120/80 mm Hg. B. Respiratory rate is 10 breaths per minute. C. Urine output is 40 ml per hour. D. Serum magnesium level is 5 mEq/L.

Magnesium sulfate is used as a tocolytic. However, it can cause severe adverse effects. Therefore the nurse should closely monitor the client. A respiratory rate of 10 breaths per minute indicates that the client has respiratory depression, which is an adverse effect of magnesium sulfate. Therefore the nurse should stop administration of the drug. A blood pressure of 120/80 mm Hg is normal and does not require discontinuation of magnesium sulfate. Urine output of 40 ml per hour indicates normal urine output; hence, the nurse need not discontinue the therapy. The therapeutic serum magnesium level should be 5 mEq/L to exert its action. Therefore if the serum magnesium level is 5 mEq/L, the nurse need not discontinue the therapy, because it would not cause toxic effects. B

A client reports painless, bright-red vaginal bleeding during the second trimester of pregnancy. Upon assessment, the nurse finds that the client's urine output has decreased, the fundal height has increased, and the uterus is nontender with normal tone. What does the nurse interpret from these findings? A. The client has placenta previa. B. The client has ectopic pregnancy. C. The client has hydatidiform mole. D. The client has normal development.

Placenta previa is an obstetric complication in which the placenta is implanted partially or completely in the lower uterine segment (near to or covering the cervix). Painless, bright-red vaginal bleeding takes place during the second trimester. Decreased urine output, greater-than-expected fundal height, and a nontender uterus with normal tone are signs of placenta previa. In ectopic pregnancy the fertilized ovum is implanted outside the uterine cavity. Hydatidiform mole is a benign proliferative growth of the placental trophoblast in which the chorionic villi develop into cystic vesicles that hang in a grapelike cluster. Decreased urine output is a complication seen in clients with diffused intravascular coagulopathy (DIC). A

A pregnant client is diagnosed with abruptio placentae. What signs and symptoms would the nurse find in the client? Select all that apply. A. Hypoglycemia B. Abdominal pain C. Vaginal bleeding D. Delayed menses E. Uterine tenderness

Premature separation of the placenta from the uterus is called as abruptio placentae. Vaginal bleeding, abdominal pain, and uterine tenderness are signs and symptoms of abruptio placentae. Abruptio placentae does not affect blood glucose level; therefore, it does not cause hypoglycemia. Delayed menses is a sign of an ectopic pregnancy. B, C, E

In caring for the woman with disseminated intravascular coagulation (DIC), what order should the nurse anticipate? A. Administration of blood B. Preparation of the woman for invasive hemodynamic monitoring C. Restriction of intravascular fluids D. Administration of steroids

Primary medical management in all cases of DIC involves correction of the underlying cause, volume replacement, blood component therapy, optimization of oxygenation and perfusion status, and continued reassessment of laboratory parameters. Central monitoring would not be ordered initially in a woman with DIC because this can contribute to more areas of bleeding. Management of DIC includes volume replacement, not volume restriction. Steroids are not indicated for the management of DIC. A

A nurse taught a pregnant woman about the importance of iron and taking her iron supplement daily. Which statement by the woman indicates the need for further instruction? A. "I take my iron supplement with a cup of tea" B. "I include a lot of citrus fruit and tomatoes in my diet" C. "My stools can turn black or dark green" D. " I need to keep track of my bowel movements because taking iron can lead to constipation.

A

A nurse is reviewing the chart of a woman being seen for a routine prenatal visit. the chart documents concerns with Couvade syndrome. The nurse understand this includes: A. The partner experiencing maternal signs and symptoms B. A history of prior problems with preterm labor C. A history of multidrug abuse in the pregnant female D. A history of extreme pregnancy-related nausea and vomiting

A

A client diagnosed with endometritis is prescribed clindamycin (Cleocin). What comfort measures does the nurse teach the client? Select all that apply. A. Avoid sitz baths. B. Use warm blankets. C. Decrease fluid intake. D. Change perineal pads. E. Avoid cool compresses.

5 Avoid cool compresses. A client diagnosed with endometritis is instructed to use warm blankets to cover the body for pain relief. The client should also be instructed to change the perineal pads, so as to prevent the spread of infection. The client is advised to change the perineal pads from front to back to maintain hygiene at the infected site. Sitz baths are encouraged to maintain hygiene and provide comfort. Fluid intake is suggested to maintain the fluidity of the blood. Applying cool compresses to the site of infection is advised to relieve discomfort caused by the infection. B, D

A 30-year-old woman at 16 weeks gestation comes for a routine prenatal visit. Her 24-hour dietary recall is evaluated by the nurse. which entry indicates that this woman needs further instructions regarding nutrient needs during pregnancy? A. Eight ounces total from the meat, poultry, fish, dry beans, eggs, and nuts group B. Three cups of vegetables and two cups of fruit C. Seven ounces of grains, including whole-grain bread and bran cereal D. Three cups from the milk, yogurt, and cheese group

A

A pregnant client reports having strange food cravings. She admits that she feels compelled to eat soil sometimes. What health risk factor does the nurse warn the client about? A. Eating soil may lead to gestational diabetes. B. Eating soil may lead to low birth weight in an infant. C. Eating soil may increase the level of lead in your blood. D. Eating soil may cause excess weight gain during pregnancy.

A client who feels compelled to eat soil or other nonfood items may be suffering from pica. This condition is particularly problematic during pregnancy, when nutrition is essential to the mother and the developing fetus. Soil may be contaminated with heavy metals and could therefore lead to elevated lead levels in the blood. Gestational diabetes is not likely to develop from eating soil, but is a risk if the client consumes cornstarch or other empty-calorie foods as part of the pica cravings. Poor nutrition is linked to low birth weight in infants, but it is not directly tied to eating soil. Eating soil is not likely to lead to excess weight gain during pregnancy, because there are no nutrients in soil to be absorbed by the body. C

A woman at 37 weeks of gestation is admitted with a placental abruption after a motor vehicle accident. Which assessment data are most indicative of her condition worsening? A. P 112, R 32, BP 108/60; FHR 166-178 B. P 98, R 22, BP 110/74; FHR 150-162 C. P 88, R 20, BP 114/70; FHR 140-158 D. P 80, R 18, BP 120/78; FHR 138-150

Bleeding, which impacts the mother's well-being as well as that of her fetus, is the most dangerous problem. The decreasing blood volume would cause increases in pulse and respirations and a decrease in blood pressure. The fetus often responds to decreased oxygenation as a result of bleeding, causing a decrease in perfusion. This causes the fetus' heart rate to increase above the normal range of 120-160 beats per minute. The other options have measurements that are in the "normal" range and would not reflect a deterioration of the patient's physical status. A

A pregnant woman and her partner are preparing her birth plan. It is the couple's wish that the woman give birth in warm water. The nurse recognizes this style of birthing as the: A. Lamaze method B. LeBoyer method C. Odent method D. Dick-Read method

C

A pregnant woman at 32 weeks of gestation comes to the emergency department because she has begun to experience bright red vaginal bleeding. She reports that she has no pain. The admission nurse suspects that the woman is experiencing: A. Abruptio placentae B. Disseminated intravascular coagulation C. Placenta previa D. Preterm labor

C

The perinatal nurse understands that a puerperal infection occurs within how many days after giving birth? A. 10 to 14 B. 15 to 30 C. Within 28 days D. Within 6 months

C

The prenatal nurse counsels a woman to stop smoking prior to conceiving. Which of the following does the nurse advise is a potential complication of smoking during pregnancy? A. Gestational diabetes mellitus B. Intellectual and developmental disabilities C. Intrauterine growth restriction D. Hyperirritability

C

Which measure is least effective in helping a woman to prevent postpartum depression? A. Share feelings and emotions with family members and her partner. B. Recognize that emotional problems after having a baby are not unusual. C. Care for the baby by herself to increase her level of self-confidence and self esteem. D. Ask friends or family members to take care of the baby while she sleeps or has a date with her partner.

C

Women with inadequate weight gain during pregnancy are at higher risk of giving birth to an infant with: A. Spina bifida. B. Intrauterine growth restriction. C. Diabetes mellitus. D. Down syndrome.

Both normal-weight and underweight women with inadequate weight gain have an increased risk of giving birth to an infant with intrauterine growth restriction. Spina bifida is not associated with inadequate maternal weight gain; an adequate amount of folic acid has been shown to reduce the incidence of this condition. Diabetes mellitus is not related to inadequate weight gain. A mother with gestational diabetes is more likely to give birth to a large-for-gestational age infant. Down syndrome is the result of trisomy 21, not inadequate maternal weight gain. B

What dose of calcium supplement should a lactating mother with calcium deficiency receive? A. 400 mg B. 500 mg C. 900 mg D. 600 mg

Calcium is an important mineral required for a lactating mother because it promotes lactation. A lactating woman should maintain adequate calcium intake or take a 600-mg calcium supplement each day. A dosage of 400 mg or 500 mg per day would not be sufficient for a lactating or pregnant woman. A dosage of 900 mg may lead to hypercalcemia. D

The nurse is caring for a pregnant client who reports constipation. Which instructions does the nurse give to the client about relieving constipation? Select all that apply. A. "Consume at least 28 g of fiber per day." B. "Eat more eggs daily." C. "Eat whole grains and fresh fruits." D. "Eat a good quantity of meat daily." E. "Drink at least 50 ml/kg/day of fluids."

Constipation generally occurs due to decreased bowel motility. Increased intake of fiber increases bowel motility. The recommended fiber intake for a pregnant patient is 28 g daily. Increased intake of fiber can be achieved by eating fiber-rich foods such as whole grains and fresh fruits. Constipation can also be relieved by drinking adequate fluids (at least 50 ml/kg/day), because fluids are absorbed in the large intestine, making stool softer and facilitating its expulsion. Consuming more eggs may not relieve constipation, because they are a source of protein and have low fiber content. Excess consumption of meat may aggravate constipation. A, C, E

A woman's last normal menstrual period (LNMP) began on November 9, 2014 and ended on November 14, 2014. Using Nagele's rule, the estimated date of birth is: A. July 2, 2015 B. July 7, 2015 C. August 16, 2015 D. August 21, 2015

D

A postpartum client who had undergone a cesarean reports to the nurse about fever, loss of appetite, pelvic pain, and foul-smelling lochia. Upon assessment, the nurse finds that the client has an increased pulse rate and uterine tenderness. The laboratory reports indicate significant leukocytosis. What clinical condition should the nurse suspect based on these findings? A. Cystocele. B. Rectocele. C. Hematoma. D. Endometritis.

Endometritis is a common postpartum infection. It usually begins as a localized infection at the placental site and spreads to the entire endometrium. Fever, loss of appetite, pelvic pain, and foul-smelling lochia are symptoms of endometritis. An increased pulse rate and uterine tenderness is also observed in this condition. Therefore the nurse can infer that the client has endometritis. Cystocele is the protrusion of the bladder downward into the vagina. Rectocele is the herniation of the anterior rectal wall through the relaxed or ruptured vaginal fascia and rectovaginal septum. The symptoms reported by the client are not indicative of these conditions. Because the nurse does not find any collection of blood in the client, the client does not have a hematoma. D

What does the nurse administer to a client if there is excessive bleeding after suction curettage? A. Nifedipine (Procardia) B. Methyldopa (Aldomet) C. Hydralazine (Apresoline) D. Ergonovine (Methergine)

Ergonovine (Methergine) is an ergot product, which is administered to contract the uterus when there is excessive bleeding after suction curettage. Nifedipine (Procardia) is prescribed for gestational hypertension or severe preeclampsia. Methyldopa (Aldomet) is an antihypertensive medication indicated for pregnant clients with hypertension. Hydralazine (Apresoline) is also an antihypertensive medication used for treating hypertension intrapartum. D

A pregnant woman demonstrates understanding of the nurse's instructions regarding relief of leg cramps if she: A. Wiggles and points her toes during the cramp. B. Applies cold compresses to the affected leg. C. Extends her leg and dorsiflexes her foot during the cramp. D. Avoids weight bearing on the affected leg during the cramp.

Extending the leg and dorsiflexing the foot are the appropriate relief measure for a leg cramp. Pointing the toes can aggravate rather than relieve the cramp. Application of heat is recommended. Bearing weight on the affected leg can help relieve the leg cramp, so it should not be avoided. C

Which statement is most likely to be associated with a breech presentation? A. Least common malpresentation B. Descent rapid C. Diagnosis by ultrasound only D. High rate of neuromuscular disorders

Fetuses with neuromuscular disorders have a higher rate of breech presentation, perhaps because they are less capable of movement within the uterus. Breech is the most common malpresentation, affecting 3% to 4% of all labors. Descent is often slow because the breech is not as good a dilating wedge as the fetal head. Diagnosis is made by abdominal palpation and vaginal examination, and is confirmed by ultrasound. D

What is the primary cause of thromboembolic disease? A. Viral infection B. Hypercoagulation C. Corticosteroid therapy D. Deficient clotting factors

Hypercoagulation and venous state are the primary causes for thromboembolic disease. Thromboembolic disease is characterized by the formation of clots in the blood vessel mainly due to inflammation. Viral infection is not associated with the formation of clots. Administration of corticosteroids does not alter the clotting behavior in a patient. Deficiency of clotting factors results in bleeding; it is not associated with thromboembolic disease. B

Which drug is used for treating a client with severe postpartum bleeding? A. Nifedipine (Adalat) B. Oxytocin (Pitocin) C. Propranolol (Inderal) D. Metronidazole (Flagyl)

Oxytocin (Pitocin) is a synthetic hormone used to induce labor and to control severe postpartum bleeding by making the uterus contract. Nifedipine (Adalat) is a calcium channel blocker that is used intocolytic therapy for preterm labor. Propranolol (Inderal) is used to reverse intolerable cardiovascular effects of terbutaline (Brethine). Metronidazole (Flagyl) is a broad-spectrum antibiotic that is used to treat chorioamnionitis after cesarean birth. B

The most prevalent clinical manifestation of abruptio placentae (as opposed to placenta previa) is what? A. Bleeding B. Intense abdominal pain C. Uterine activity D. Cramping

Pain is absent with placenta previa but may be agonizing with abruptio placentae. Bleeding may be present in varying degrees for both placental conditions. Uterine activity may be present with both placental conditions. Cramping is a form of uterine activity that may be present in both placental conditions. B

Which ratio would be used to restore effective circulating volume in a postpartum patient who is experiencing hypovolemic shock? A. 4:1 B. 2:1 C. 1:1 D. 3:1

Patient complaint of a headache accompanied by an increased reaction (response) time indicates that cerebral hypoxia is getting worse. Return of blood pressure to normal range would indicate resolving symptoms. Brisk capillary refill is a normal finding. The patient may see "stars" early on in decreased blood flow states. D

Some men experience pregnancy-like symptoms, such as nausea, weight gain, and other physical symptoms. What is this phenomenon called? A. Quickening B. Labor syndrome C. Couvade syndrome D. Pregnancy syndrome

The phenomenon in which men experience pregnancy-like symptoms, such as nausea, weight gain, and other physical symptoms is called couvade syndrome. Quickening is the mother's first perception of fetal movement. Labor syndrome and pregnancy syndrome are not part of this phenomenon. C

On assessing the laboratory reports of a client who is 12 weeks pregnant, the nurse observes that the client's level of serum ferritin is low. Which condition does the nurse expect in the client? A. Tetany B. Anemia C. Renal failure D. Hypertension

Women are at an increased risk for iron deficiency during pregnancy. Iron is needed to allow the transfer of adequate iron to the fetus and for the expansion of the maternal red blood cell (RBC) mass. The serum ferritin level is an indicator of iron content in the body. Poor iron status results in iron-deficiency anemia. Tetany occurs due to calcium deficiency. Renal failure may occur due to an imbalance of electrolytes such as potassium and sodium. Hypertension may occur due to high intake of sodium and potassium. B

In planning for an expected cesarean birth for a woman who has given birth by cesarean previously and who has a fetus in the transverse presentation, the nurse includes which information? A. "Because this is a repeat procedure, you are at the lowest risk for complications." B. "Even though this is your second cesarean birth, you may wish to review the preoperative and postoperative procedures." C. "Because this is your second cesarean birth, you will recover faster." D. "You will not need preoperative teaching because this is your second cesarean birth."

"Even though this is your second cesarean birth, you may wish to review the preoperative and postoperative procedures." is the most appropriate response. "Because this is a repeat procedure, you are at the lowest risk for complications." is not accurate. Maternal and fetal risks are associated with every cesarean section. "Because this is your second cesarean birth, you will recover faster." is not accurate. Physiologic and psychologic recovery from a cesarean section are multifactorial and individual to each client each time. Preoperative teaching should always be performed regardless of whether the client has already had this procedure. B

A nurse is reviewing hypotonic labor with a student nurse. the nurse enplanes to the student that which of the following is the most common cause of this dysfunctional labor pattern? A. Fetal macrosomia B. Maternal android pelvis C. Inadequate uterine pacemakers D. Fetal occiput-posterior position

A

A perinatal nurse screens all patients for intimate partner violence. What technique is best when preforming this screening? A. Asking direct questions about abuse B. Having the woman fill out a survey C. Scheduling a social worker to visit all new patients D. Distributing flyers that encourage reporting abuse

A

A pregnant woman at 38 weeks of gestation and diagnosed with marginal placenta previa has just been given birth to a healthy newborn male. The nurse recognizes that the immediate focus for the care of this woman is: A. Preventing hemorrhage B. Relieving acute pain C. Preventing infection D. Fostering attachment of the woman with her new son.

A

A primigravida at 10 weeks of gestation reports mild uterine cramping and slight vaginal spotting without passage of tissue. When she is examined, no cervical dilation is noted. The nurse caring for this woman should: A. Anticipate that the woman will be sent home with instructions to limit her activity and to avoid stress or orgasm. B. Prepare the woman for a dilation and curettage. C. Notify a grief counselor to assist the woman with the imminent loss of her fetus. D. Tell the woman that the doctor will most likely perform a cerclage to help her maintain her pregnancy

A

A priority question to ask a woman experiencing postpartum depression is: A. "Have you thought about hurting yourself?" B. "How often do you cry" C. "Have you been feeling insecure, fragile, or vulnerable?" D. "Does the responsibility of motherhood seem overwhelming?"

A

The nurse is caring for a postpartum patients understands that which of the following is the most common type of psychosocial disturbance seen in this population? A. Postpartum blues B. Postpartum depression C. Postpartum psychosis D. Postpartum mania

A

The nurse responsible for the care of postpartum women recognizes that the first sign of puerperal infection most likely is: A. A temperature elevation to 38*C or higher after the first 24 hours following birth B. Increased white blood cell count C. Foul smelling profuse lochia D. Bradycardia

A

The perinatal nurse is aware that complications arising from amnioinfusion include which of the following? A. Infection B. Halt in labor C. Neonatal hydrocephalus D. Fluid overload

A

The perinatal nurse knows that which of the following conditions must be met before assisting at forceps delivery? A. Presenting part must be engaged B. Membranes must be intact C. Patient's bladder should be full D. Cervix at least 50% dilated.

A

The perinatal nurse promotes postpartum health and prevents infection with the inclusion of information about which concept? A. Good hand washing B. Early ambulation C. Minimal fluid intake D. Restricted protein intake

A

The perinatal nurse understands that one of the risks of oxytocin infusion includes FHR changes related to which of the following? A. Decreased placental perfusion B. Oligohydramnios C. Maternal hypotonic contractions D. Maternal hypotension

A

A pregnant woman with a body mass index (BMI) of 22 asks the nurse how she should be gaining weight during pregnancy. The nurse's best response is to tell the woman that her pattern of weight gain should be approximately: A. a pound a week throughout pregnancy. B. 2 to 5 lbs during the first trimester, then a pound each week until the end of pregnancy. C. a pound a week during the first two trimesters, then 2 lbs per week during the third trimester. D. a total of 25 to 35 lbs.

A BMI of 22 represents a normal weight. Therefore, a total weight gain for pregnancy is about 25 to 35 lbs or about 2 to 5 lbs in the first trimester and about 1 lb per week during the second and third trimesters. One pound per week is not the correct guideline during pregnancy. One pound per week during the first two trimesters and two pounds per week thereafter is not the correct guideline for weight gain during pregnancy. A total weight gain of 25 to 35 pounds is correct, but the pattern needs to be explained to the woman. B

The nurse is caring for a pregnant client whose prepregnant body mass index (BMI) is 23.5. Under which BMI category does the nurse categorize the patient? A. Obese B. Normal C. Overweight D. Underweight

A commonly used method of evaluating the appropriateness of weight in terms of height is the body mass index (BMI). BMI is calculated by using the formula: BMI = weight/height 2. In this formula, the weight is in kilograms and the height is in meters. Prepregnant BMI can be classified as normal if the BMI is in the range of 18.5 to 24.9. If it is less than 18.5, it is classified as underweight or low BMI. If the BMI ranges from 25 to 29.9, it is classified as overweight or high BMI, and if it is greater than 30, it is classified as obese BMI. Because the client's BMI lies between 18.5 and 24.9, the client has normal BMI. The client is not obese, overweight, or underweight. B

What type of cultural concern is the most likely deterrent to many women seeking prenatal care? A. Religion B. Modesty C. Ignorance D. Belief that physicians are evil

A concern for modesty is a strong deterrent to many women seeking prenatal care; for some women, exposing body parts, especially to a man, is considered a major violation of modesty. There are other deterrents. Even if the prenatal care described is familiar to a woman, some practices may conflict with the beliefs and practices of a subculture group or religion to which she belongs. For many cultural groups a physician is deemed appropriate only in times of illness. Because pregnancy is considered a normal process and the woman is in a state of health, the services of a physician are considered inappropriate. Many cultural variations are found in prenatal care, so ignorance is not likely to be a deterrent to women seeking prenatal care. B

With regard to the father's acceptance of the pregnancy and preparation for childbirth, the maternity nurse should know that: A. The father goes through three phases of acceptance of his own. B. The father's attachment to the fetus cannot be as strong as that of the mother because it does not start until after birth. C. In the last 2 months of pregnancy, most expectant fathers suddenly get very protective of their established lifestyle and resist making changes to the home. D. Typically men remain ambivalent about fatherhood right up to the birth of their child.

A father typically goes through three phases of acceptance: accepting the biologic fact, adjusting to the reality, and focusing on his role. Typically, the expectant father's ambivalence ends by the first trimester, and he progresses to adjusting to the reality of the situation and focusing on his role. The father-child attachment can be as strong as the mother-child relationship and can begin during pregnancy. In the last 2 months of pregnancy, many expectant fathers work hard to improve the environment of the home for the child. A

A nurse is caring for a client whose labor is being augmented with oxytocin. The nurse recognizes that the oxytocin should be discontinued immediately if there is evidence of: A. Uterine contractions occurring every 8 to 10 minutes. B. A fetal heart rate (FHR) of 180 with absence of variability. C. The client needing to void. D. Rupture of the client's amniotic membranes.

A fetal heart rate (FHR) of 180 with absence of variability is nonreassuring; the oxytocin should be immediately discontinued and the physician should be notified. The oxytocin should also be discontinued if uterine hyperstimulation occurs. Uterine contractions that occur every 8 to 10 minutes do not qualify as hyperstimulation. The client needing to void is not an indication to discontinue the oxytocin induction immediately or to call the physician. The oxytocin does not need to be discontinued when the membranes rupture, but the physician should be notified. B

With regard to nutritional needs during lactation, a maternity nurse should be aware that: A. The mother's intake of vitamin C, zinc, and protein now can be lower than during pregnancy. B. Caffeine consumed by the mother accumulates in the infant, who therefore may be unusually active and wakeful. C. Critical iron and folic acid levels must be maintained. D. Lactating women can go back to their prepregnant calorie intake.

A lactating woman needs to avoid consuming too much caffeine. Vitamin C, zinc, and protein levels need to be moderately higher during lactation than during pregnancy. The recommendations for iron and folic acid are somewhat lower during lactation. Lactating women should consume about 500 kcal more than their prepregnancy intake, at least 1800 kcal daily overall. B

Upon reviewing the ultrasonography reports of a pregnant client, the nurse finds that the placenta is at a distance of 2.5 cm from the internal cervical os. What complication is likely if the client has a vaginal delivery? A. Hemorrhage B. Hyperthyroidism C. Thrombocytopenia D. Hypofibrinogenemia

A placenta implanted in the lower uterine segment 2.5 cm from the internal cervical os indicates that the client has marginal placenta previa. In placenta previa, disruption of placental blood vessels occurs with stretching and thinning of the lower uterine segment, which results in bleeding. Therefore, the major maternal complication associated with placenta previa is hemorrhage. Hyperthyroidism is one of the serious complications of hydatidiform mole. Thrombocytopenia and hypofibrinogenemia are complications of abruptio placentae. A

The nurse is preparing to discharge a 30-year-old woman who has experienced a miscarriage at 10 weeks of gestation. Which statement by the woman indicates a correct understanding of the discharge instructions? A. "I will not experience mood swings, because I was only at 10 weeks of gestation." B. "I will avoid sexual intercourse for 6 weeks and pregnancy for 6 months." C. "I should eat foods that are high in iron and protein to help my body heal." D. "I should expect the bleeding to be heavy and bright red for at least 1 week."

A woman who has experienced a miscarriage should be advised to eat foods that are high in iron and protein to help replenish her body after the loss. After a miscarriage a woman may experience mood swings and depression from the reduction of hormones and the grieving process. Sexual intercourse should be avoided for 2 weeks or until the bleeding has stopped, and pregnancy should be avoided for 2 months. The woman should not experience bright red, heavy, profuse bleeding; this should be reported to the health care provider. C

During an early-bird prenatal class, a nurse teaches a group of newly diagnosed pregnant women about their emotional reactions during pregnancy. What topics should the nurse discuss with the woman? (circle all that apply) A. Sexual desire (libido) usually increases during the second trimester of pregnancy. B. A referral for counseling should be sought if a woman experiences conflicting feelings about her pregnancy in the first trimester. C. Rapid, unpredictable mood swings reflect gestational bipolar disorder. D. A quiet period of introspection is often experienced around the same time a woman feels her baby move for the first time. E. A woman's own mother is usually her greatest source of emotional support during pregnancy. F. Attachment to her baby begins late in the third trimester, when she begins attending childbirth preparation classes and realizes that the baby will arrive soon.

A, D

A nurse teaching a pregnant woman about the importance of iron in her diet would tell her to consume which foods as a good source of iron? (Circle all that apply) A. Whole-grain breads and cereals B. Oranges C. Salmon D. Raisins E. Spinach F. Tomatoes

A, D, E

A woman at 27 weeks of gestation experiences some mild uterine cramping. Which actions should she take? (Circle all that apply.) A. Empty her bladder B. Call her nurse-midwife immediately. C. Relax in a chair D. Drink two to three glasses of water or juice. E. Palpate her uterus for 1 hour F. Resume the activity she was doing if the cramping subsides.

A, D, E

The nurse evaluates a pregnant woman's knowledge about prevention of urinary tract infections (UTIs) at the prenatal visit following a class on infection prevention that the woman attended. The nurse recognizes that the woman needs further instruction when she tells the nurse about which measures that she now uses to prevent urinary tract infections. (Circle all that apply) A. "I drink about one quart of fluid a day" B. "I have stopped using bubble baths and bath oils" C. "I have started wearing panty hose and underpants with a cotton crotch." D. "I have intercourse with my husband only once a week now because it could lead to bladder infections" E "If I drink cranberry juice at least twice a day, I will not get an infection"

A, D, E

A client who has undergone cesarean surgery reports to the nurse about having persistent perineal pain and feels pressure in the vagina. The nurse finds that the client is in shock. What clinical condition should the nurse suspect based on this assessment? A. Rectocele. B. Endometritis. C. Impaired lactation. D. Retroperitoneal hematoma.

Accumulation of blood in the retroperitoneal space is called retroperitoneal hematoma. It is caused by the rupture of the cesarean scar during labor. Retroperitoneal hematoma is characterized by such symptoms as persistent perineal pain, a feeling of pressure in the vagina, and shock. Therefore it is evident that the client has this condition. Persistent perineal pain, a feeling of pressure in the vagina, and shock are not associated with rectocele, endometritis, and impaired lactation. Rectocele is the herniation of the anterior rectal wall through the relaxed or ruptured vaginal fascia and rectovaginal septum. Endometritis is characterized by fever, increased pulse rate, chills, anorexia, nausea, fatigue, pelvic pain, uterine tenderness, and foul-smelling lochia. Because the client did not report these symptoms, the client does not have endometritis. Perineal pain, a feeling of pressure in the vagina, and shock do not affect lactation, so the client does not have impaired lactation. D

The nurse is caring for a newborn after a vacuum-assisted birth. What changes should the nurse monitor in the newborn? Select all that apply. A. Inability to pass urine B. Yellow discoloration of skin C. Listlessness D. Poor sucking patterns E. Difficulty breathing

After a vacuum-assisted birth, the newborn might be at the risk of hyperbilirubinemia as the bruising resolves, which may cause neonatal jaundice. So, the nurse should monitor the newborn for yellow discoloration of the skin. A vacuum-assisted birth may cause cerebral irritation in the newborn, which manifests as listlessness and poor sucking. Inability to pass urine may indicate structural anomalies and may not be due to vacuum delivery. Difficulty in breathing can be caused by many factors and not necessarily due to vacuum delivery. B, C, D

A postpartum woman with asthma who had manual repositioning of her uterus into the pelvic cavity after uterine inversion, experiences prolonged lochial discharge and hemorrhage. Upon examination, the nurse finds a large, boggy uterus. Which nursing intervention may pose the most risk to the client? A. Removal of retained placental fragments B. Administration of carboprost (Hemabate) C. Aggressive massage of the uterine fundus D. Administration of intravenous oxytocin (Pitocin)

Aggressive fundal massage should be avoided in a client who underwent a manual repositioning of the uterus, because this may increase the risk of bleeding. Subinvolution of the uterus is the delayed return of the uterus to its normal size and function. Subinvolution of the uterus is characterized by prolonged lochial discharge, excessive bleeding, hemorrhage, and a large, boggy uterus. Retained placental fragments should be removed by performing dilation and curettage. This may reduce excessive bleeding. Carboprost (Hemabate), a uterotonic medication, is used if the uterus is nonresponsive to the continuous oxytocin (Pitocin) infusion. Carboprost (Hemabate) is contraindicated in clients with asthma. Because the client has asthma, carboprost (Hemabate) should not be administered to the client. Administration of intravenous oxytocin (Pitocin) along with normal saline promotes restoration of normal uterine tone and size. C

A woman who is 14 weeks pregnant tells the nurse that she always had a glass of wine with dinner before she became pregnant. She has abstained during her first trimester and would like to know whether it is safe for her to have a drink with dinner now. The nurse tells her: A. "Because you're in your second trimester, there's no problem with having one drink with dinner." B. "One drink every night is too much. One drink three times a week should be fine." C. "Because you're in your second trimester, you can drink as much as you like." D. "Because no one knows how much or how little alcohol it takes to cause fetal problems, the best course is to abstain throughout your pregnancy."

Although the consumption of occasional alcoholic beverages may not be harmful to the mother or her developing fetus, complete abstinence is strongly advised. A safe level of alcohol consumption during pregnancy has not yet been established. The first trimester is a crucial period of fetal development, but pregnant women at all gestations are counseled to eliminate all alcohol from their diet. D

The nurse is monitoring a pregnant client after amniotomy. Which observation would indicate a likelihood of umbilical cord compression? A. The fetal heart rate (FHR) confirms tachycardia. B. The client's vaginal drainage has a foul-smell. C. The client has maternal chills frequently. D. The fetal heart rate (FHR) has variable decelerations.

Amniotomy is performed in a pregnant client in order to rupture the membranes artificially. After the procedure, the nurse should closely monitor the FHR. Reduced FHR and variable decelerations in FHR indicate that the client's umbilical cord is compressed. The nurse should immediately inform the primary health care provider of the client's condition. Tachycardia or increased FHR are common manifestations observed after amniotomy. Tachycardia does not require immediate clinical action. Maternal chills and foul-smelling vaginal discharge after amniotomy indicate infection of the ruptured membranes. However, this would not be a reason to expect umbilical cord compression. D

A woman who is 32 weeks pregnant is informed by the nurse that a danger sign of pregnancy could be: A. Constipation B. Alteration in the pattern of fetal movement C. Heart palpitations D. Edema in the ankles and feet at the end of the day

An alteration in the pattern or amount of fetal movement may indicate fetal jeopardy. Constipation is a normal discomfort of pregnancy that occurs in the second and third trimesters. Heart palpitations are a normal change related to pregnancy; they are most likely to occur during the second and third trimesters. As the pregnancy progresses, edema in the ankles and feet at the end of the day is not uncommon. B

Upon assessment the nurse finds that the client who has undergone a cesarean delivery is at risk of postpartum hemorrhage (PPH). What would be the most likely cause of PPH in this client? A. Anesthesia B. Coagulopathy C. Placenta previa D. Chorioamnionitis

Anesthesia blocks the neurologic impulses that stimulate uterine contractions (UCs). This causes uterine atony and can lead to PPH. Chorioamnionitis is a serious condition in which the fetal membranes are infected, and is associated with other signs, such as fever. Coagulopathy is a clotting disorder, which occurs due to protein defects in the plasma that interrupt the coagulation cascade and cause blood coagulation. Placenta previa is an obstetric complication in which the placenta penetrates partially in the lower uterine segment. This is a common complication in pregnant women who smoke and consume cocaine. A

The nurse is teaching a group of pregnant clients about early identification of preterm labor. What signs and symptoms of preterm labor should the nurse include in the teaching? Select all that apply. A. Upper abdominal pain B. Increased vaginal discharge C. Presence of vaginal bleeding D. Decreased urinary frequency E. Painful uterine contractions (UCs)

Any pregnant client runs the risk of having preterm labor and should be educated to identify its signs and symptoms. Painful uterine contractions (UCs) are a sign of preterm labor, caused by the body's attempt to deliver the baby. The client may show signs of vaginal bleeding due to a rupture of the membranes. Preterm labor can also be identified by changes in the color or amount of vaginal discharge. During labor the vaginal discharge usually increases and becomes brown to red in color. Preterm labor is also characterized by an increase in urine frequency and pain in lower abdomen. Therefore a decrease in urine frequency and upper abdominal pain do not indicate preterm labor. B, C, E

Which medication is contraindicated in a client who is on anticoagulant therapy? A. Aspirin (Ecotrin) B. Clindamycin (Cleocin) C. Misoprostol (Cytotec) D. Ergonovine (Ergotrate)

Aspirin (Ecotrin) is contraindicated in clients undergoing anticoagulant therapy, because it inhibits the synthesis of clotting factors and can lead to prolonged clotting time. Clindamycin (Cleocin) does not inhibit the synthesis of clotting factors and can be administered in clients undergoing anticoagulant therapy. Misoprostol (Cytotec) and ergonovine (Ergotrate) do not affect the clotting factors and do not interact with anticoagulants. Thus, it is safe to administer misoprostol (Cytotec) and ergonovine (Ergotrate) to clients undergoing anticoagulant therapy. A

A 25-year-old pregnant woman is at 10 weeks of gestation. Her BMI is calculated to be 24. Regarding weight gain during pregnancy, the nurse should recommend: A. A total weight gain of 18 kg B. First-trimester weight gain of 1 to 2 kg C. Weight gain of 0.4 kg each week for 40 weeks D. Weight gain of 3 kg per month during the second and third trimesters

B

A nurse hears a health-care provider describe a pregnant woman as having tocophobia. What does the nurse understand this to mean? A. Allergy to tocolytics B. Fear of childbirth C. Fear of pain D. Atonic uterus

B

A nurse reads in a woman's chart that she has a history of dystocia. Based on this information, the nurse assess the woman for what condition? A. Fetal abnormalities B. Long, difficult labor C. Prior fetal demise D. Bleeding abnormalities

B

A nurse reads on the patient's chart that she has a clot in her superficial saphenous venous system. What condition should the nurse be prepared to treat? A. Pulmonary embolism B. Superficial venous thrombosis C. Deep venous thrombosis D. Uterine thrombophlebitis

B

A patient has an order for a prostaglandin E2 preparation. What does the nurse understand about this medication? A. Only used when delivery is imminent B. Cervical ripening agent C. Has a high rate of adverse reactions D. Is only given subcutaneously

B

A postpartum woman in the fourth stage of labor received prostoglandin F2alpha (Hemabate) 0.25 mg intramuscularly. The expected outcome of care for the administration of this medication is: A. Relief from the pain of the uterine cramping B. Prevention of intrauterine infection C. Reduction in the blood's ability to clot D. Limitation of excessive blood loss that is occurring after birth

B

As part of a postpartum woman's assessment, the perinatal nurse observes for signs and symptoms of hematoma formation. Which of the following is the most common anatomical site for a hematoma to form? A Rectum B. Vulva C. Cervix D. Episiotomy site

B

Methylergonovine (Methergine) 0,2 mg is ordered for a woman who gave birth vaginally 1 hour ago; it is to be administered intramuscularly to treat a profuse lochial flow with clots. Her fundus is boggy and does not respond well to massage. She is still being treated for preeclampsia with IV magnesium sulfate at 1 g/hour. Her blood pressure (BP), measured 5 minutes ago, was 155/98 mm Hg. In fulfilling this order, the nurse should: A. Measure the woman's blood pressure again 5 minutes after administering the medication B. Question the order, based on the woman's hypertensive status C. Administer the methylergonovine because it is best choice to counteract the possible uterine relaxation effects of the magnesium sulfate infusion the woman is receiving D. Tell the woman that the medication will lead to uterine cramping

B

The nurse should recognize that a complication of pregnancy associated with the intravenous use of cocaine is: A. Prolonged, difficult labor B. Premature separation of the placenta C. Increased risk for vaginal and urinary tract infections D. Severe fetal/ neonatal central nervous system (CNS) depression)

B

The perinatal nurse is providing information to a postpartum woman being discharged from the hospital on warfarin (Coumadin) therapy. Which drug would the nurse instruct the patient to restrict? A. Acetaminophen (Tylenol) B. Ibuprofen (Motrin) C. Prenatal vitamins D. Docusate sodium (Colace)

B

What teaching is important for the woman being treated with lithium for postpartum psychosis? A. Dental visits every 4 months B. Lithium levels drawn every 6 months D. Do not drink any citrus juices C.Use two types of birth control

B

A woman diagnosed with an ectopic pregnancy is to receive methotrexate. The nurse should explain to the woman that: (circle all that apply) A. Methotrexate is an analgesic that will relieve the dull abdominal pain she is experiencing B. She should double-flush the toilet with the lid down for 72 hours after receiving methotrexate. C. She will receive the medication intramuscularly D. She must stop taking folic acid supplements as long as she is on methotrexate E. Her partner should use a condom during intercourse F. She must return weekly for a measurement of her progesterone level to determine if the methotrexate therapy has been effective

B, C, D

A woman demonstrates an understanding of the importance of increasing her intake of foods that are good sources of folic acid (50 mcg or more) when she includes which foods in her diet? (Circle all that apply) A. Baked haddock B. Lentil soup C. scrambled eggs D. Fruit salad of papaya and oranges E. steamed asparagus F. Corn on the cob

B, D, E

A physician has ordered that dinoprostone (Cervidil) be administered to ripen a pregnant woman's cervix in preparation for an induct labor. In fulfilling this order, the nurse should: A.Insert the dinoprostone in the cervical canal just below the internal os B. Tell the woman to remain in bed for at least 15 minutes C. Observe the woman for signs of uterine tachysystole D. Remove the dinoprostone as soon as the woman begins to experience uterine contractions

C

Which priority action would be most beneficial in helping a couple deal with fetal loss following the delivery of a stillborn? A. Allow all family members to come in immediately after the delivery to console the couple. B. Provide a quiet environment for the couple for several hours restricting any visitors or family members. C. Take a photograph of the stillborn prior to the patient's discharge to use as a keepsake. D. Allow the parents to hold and view the baby following delivery if they so request.

Bonding with the stillborn by holding and viewing after delivery is well documented by research to provide a source of comfort and closure. Although it will be important for family members to comfort the couple, it is more important for the family unit to be alone to adapt to the delivery. Providing a quiet environment is important but it not the priority action to be taken at this time. Taking a photograph is important as a keepsake but it is typically taken before the stillborn leaves the hospital. D

Women with an inadequate weight gain during pregnancy are at higher risk of giving birth to an infant with what? A. Spina bifida B. Intrauterine growth restriction C. Diabetes mellitus D. Down syndrome

Both normal-weight and underweight women with inadequate weight gain have an increased risk of giving birth to an infant with intrauterine growth restriction. Spina bifida is not associated with inadequate maternal weight gain. An adequate amount of folic acid has been shown to reduce the incidence of this condition. Diabetes mellitus is not related to inadequate weight gain. A gestational diabetic mother is more likely to give birth to a large-for-gestational age infant. Down syndrome is the result of a trisomy 21, not inadequate maternal weight gain. B

A 30-year-old woman with a BMI of 31 asks the woman about recommendations for diet and weight gain during pregnancy. What should the nurse tell this woman? A. Counsel her to begin a lifestyle change for weight reduction. B. Recommend a total weight gain goal of 4 kg during pregnancy C. Set a weight gain goal of 0.2 kg per week during the second and third trimesters D. Limit her third-trimester calorie increase to no more than 600 kcal more than prepreagnant needs.

C

A nurse is caring for a patient on IV heparin for a deep vein thrombosis. Which lab value should the nurse monitor as the priority? A. Hemoglobin B. INR C. PTT D. Platelet count

C

A pregnant woman at 6 weeks of gestation tells her nurse-midwife that she has been experiencing nausea with occasional vomiting every day. As an effective relief measure, the nurse should recommend: A. Eating starchy foods such as buttered popcorn or peanut butter with crackers in the morning before getting out of bed. B. Avoid eating before going to bed at night C. Altering eating patterns to small meals every 2 to 3 hours. D. Skipping a meal if nauseated is experienced

C

A woman has a history of a bilobed placenta, each with its own circulation. what condition does this describe? A. Battledore placenta B. Circumvallate placenta C. Succenturiate placenta D. Placenta Percreta

C

A woman has a history of placenta increta. What does the nurse understand about this condition? A. slight penetration of the myometrium by the trophoblast B. Placental perforation of the uterus C. Deep placental penetration of the myometrium D. Abnormal implantation of the placenta.

C

A woman in her second trimester calls the perinatal clinic and reports having two large cavities that the dentist wants to fill. Which response by the nurse is best? A. "You should wait until your third trimester to have dental work done". B. "It is safe to use most local anesthetics for dental work during pregnancy". C. "You will need to wait until after you have given birth to your baby." D. "You can have them filled, but you will have to avoid the anesthetics".

C

A woman is admitted through the emergency department with a medical diagnosis of ruptured ectopic pregnancy. The primary nursing diagnosis at this time is: A. Acute pain related to irritation of the peritoneum with blood B. Risk for infection related to tissue trauma C. Deficient fluid volume related to blood loss associated with rupture of the uterine tube D. Anticipatory grieving related to unexpected pregnancy outcome.

C

A woman is in active labor. On spontaneous rupture of her membranes, the nurse caring for this woman notices variable deceleration patterns during evaluation of the monitor tracing. When preparing to perform a vaginal examination, the nurse observes a small section of the umbilical cord protruding from the vagina. what should the nurse do next? A. Increase the IV drip rate B. Administer oxygen to the mother via mask at 8 - 10 L/minute C. Place a sterile gloved hand into the vagina and hold the presenting part off the cord while calling for assistance. D. Wrap the cord loosely with a sterile towel saturated with warm normal saline.

C

The perinatal nurse knows that the most ideal time to address issues related to a poor outcome in a past pregnancy is: A. Postpartum B. Prenatally C. Preconception. D. Interconception.

C

A breast feeding woman's cesarean birth occurred 2 days ago. Investigation of the pain, tenderness, and swelling in her left leg led to a medical diagnosis of deep vein thrombosis (DVT). Care management for this woman during the acute stage of the DVT involves: (Circle all that apply) A. Explaining that she will need to stop breastfeeding until anticoagulation therapy is completed B.Administering warfarin (coumadin) orally C. Placing the woman on bed rest with her left leg elevated D. Fitting the woman with an elastic stocking so that she can exercise her legs E. Telling her to avoid changing her position for the first 24 hours. F. administering heparin intravenously for 3 to 5 days

C, F

A woman at 26 weeks of gestation is being assessed to determine whether she is experiencing preterm labor. What finding indicates that preterm labor is occurring? A. Estriol is not found in maternal saliva. B. Irregular, mild uterine contractions are occurring every 12 to 15 minutes. C. Fetal fibronectin is present in vaginal secretions. D. The cervix is effacing and dilated to 2 cm.

Cervical changes such as shortened endocervical length, effacement, and dilation are predictors of imminent preterm labor. Changes in the cervix accompanied by regular contractions indicate labor at any gestation. Estriol is a form of estrogen produced by the fetus that is present in plasma at 9 weeks of gestation. Levels of salivary estriol have been shown to increase before preterm birth. Irregular, mild contractions that do not cause cervical change are not considered a threat. The presence of fetal fibronectin in vaginal secretions between 24 and 36 weeks of gestation could predict preterm labor, but it has only a 20% to 40% positive predictive value. Of more importance are other physiologic clues of preterm labor, such as cervical changes. D

A postpartum client with hemorrhagic shock is administered intravascular colloids. The nurse monitors the client carefully throughout the colloid administration. What risk factor in the client should the nurse be aware of? A. Excessive hemorrhage B. Von Willebrand disease C. Deep venous thrombosis D. Intravascular fluid overload

Clients who are given intravascular colloids are at a higher risk for intravascular fluid overload. Therefore, the nurse should monitor the client for symptoms of intravascular fluid overload. Intravascular colloid therapy does not cause excessive hemorrhage. Von Willebrand disease, a hereditary disease condition, is a type of hemophilia. Von Willebrand disease is caused by the deficiency of a blood clotting protein called Von Willebrand factor (vWF). Deep venous thrombosis is a venous thromboembolic disorder that is most commonly seen in the lower extremities. It is unrelated to intravascular colloid therapy. D

What are the risk factors associated with preterm premature rupture of membranes (PROM)? Select all that apply. A. Preeclampsia B. Long cervical length C. Cigarette smoking D. Urinary tract infection E. Uterine overdistention

Conditions such as smoking, urinary tract infection, and uterine overdistention may cause early rupturing of membranes in a pregnant patient. Therefore, these factors are considered risk factors associated with preterm PROM. Preeclampsia is the common cause of indicated preterm birth and is not associated with preterm PROM. Short cervical length would increase the risk of PROM and may not be observed in clients with long cervical length. Non-Caucasian women are at a higher risk for spontaneous preterm birth than Caucasian women. C, D, E

*What common complications are seen during the third trimester of pregnancy? Select all that apply.* Constipation Urinary frequency Disturbance in sleep Difficulty breathing Nausea and vomiting

Constipation, urinary frequency, disturbance in sleep, and difficulty breathing are the common complications that can be seen during the third trimester of pregnancy. Urinary frequency is due to the bulky uterus exerting pressure on the urinary bladder. The pregnant client may not feel comfortable while sleeping due to her large abdomen. In addition, vigorous fetal movements may disturb her sleep. The large uterus prevents lung expansion, resulting in difficulty in breathing. Nausea and vomiting usually occur in the first trimester of pregnancy. A, B, C, D

A nurse caring for a pregnant woman at 30 weeks of gestation in preterm labor. The woman's physician orders betamethasone 12 mg IV for two doses, with the first dose to begin at 11 am. In implementing this order the nurse should: A. Consult the physician, because the dose is too high B. Explain to the woman that this medication will reduce her heart rate and help her to breath easier C. Prepare to administer the medication intravenously between contractions D. Schedule the second dose for 11 am on the next day.

D

A pregnant woman is concerned about the possibility of her baby being born with a neural tube defect (NTD). Which response by the nurse is best? A. "I can ask the physician to order genetic screening for you if you like" B. "Be sure to take your iron supplement each day with a glass of orange juice". C. "you need to be sure to get at least 1,000 mg of calcium and 600 IU of vitamin D daily. D. About 70% of NTDs can be prevented by getting 400 mcg of folic acid each day.

D

A woman at 30 weeks gestation assumes a supine position for a fundal measurement and Leopold maneuvers. She begins to complain about feeling dizzy and nauseated. Her skin feels damp and cool. The nurse's first action is to: A. Assess the woman's blood pressure and pulse B. Provide the woman with an emesis basin C. Elevate the woman's legs 20 degrees from her hips D. Turn the woman on her left side

D

Doulas are becoming important members of a laboring woman's health care team. Which activity should be expected as a part of the doula's responsibilities? A. Monitoring hydration of the laboring woman, including adjustment intravenous (IV) flow rates B. Interpreting electronic fetal monitoring tracings to determine the well-being of the maternal-fetal unit. C. Eliminating the need for the husband or partner to be present during labor and birth. D. Providing continuous support throughout labor and birth, including explanations of labor process.

D

The perinatal nurse is aware of the physical effects that maternal battering can have on a newborn. Which of the following is inconsistent with this knowledge? A. Low-birth-weight infant B. Preterm birth C. Neonatal death D. Prolonged labor

D

The perinatal nurse is aware that recommendations for elective deliveries specify induction no earlier than what gestational age? A. 30 weeks B. 35 weeks C. 37 weeks D. 39 weeks

D

When conducting a health history interview during a pregnant woman's first prenatal visit, the nurse must determine if the woman is substance dependent. The nurse's first question should relate to the woman's use of: A. Alcohol B. Tobacco C. Cocaine D. Over-the-counter and prescription medications

D

The nurse finds that a client in labor has developed spontaneous bruises over the skin. The fetal heart activity monitor indicates late decelerations in the fetal heart rate (FHR). What is the best intervention to prevent fetal complications, if the nurse suspects disseminated intravascular coagulation (DIC) in the client? A. Assisting the client to supine position B. Increasing the rate of oxytocin intravenous (IV) infusion C. Administering oxygen through a face mask D. Administering intravenous (IV) fluids to the client

DIC may be caused secondary to placental abruption, which may result in late deceleration of the FHR. Late deceleration is due to decreased blood flow and oxygen to the fetus during the uterine contractions (UCs). Therefore, an oxygen mask is applied to the client. A side-lying tilt would provide proper blood flow by decreasing the pressure on the uterus. Hence, the client should be assisted to a side-lying position and not to a supine position. An IV oxytocin infusion is given to increase the UCs during labor, but it is discontinued when a late deceleration is noted. IV fluids are administered to the client to promote fluid balance, but it may not directly help in improving the fetal heart rate. C

The nurse is caring for a client who is undergoing treatment for deep vein thrombosis (DVT). Which signs and symptoms will the nurse monitor in the client to evaluate the patient's response to treatment? A. Dysuria, petechiae, and vertigo B. Petechiae, hematuria, and dysuria C. Hematuria, increased lochia, and vertigo D. Hematuria, petechiae, and increased lochia

DVT treatment includes anticoagulant therapy. The effectiveness of the treatment can be evaluated by the presence of bleeding. Hematuria is the presence of blood in the urine, which is a side effect of anticoagulant therapy. Generalized petechiae refer to the reddish patches on the skin as a result of the rupturing of the microcirculation beneath the skin. It, too, is an adverse effect associated with anticoagulant treatment. Increased lochial discharge refers to increased uterine bleeding following delivery, and is a result of excess anticoagulants. Dysuria is decreased urinary output, which is not a complication associated with anticoagulant therapy. Vertigo is not a side effect of anticoagulant therapy. D

What is the treatment that should be considered first for the client with von Willebrand disease who experiences a postpartum hemorrhage? A. Cryoprecipitate B. Factor VIII and vWf C. Desmopressin D. Hemabate

Desmopressin is the primary treatment of choice. This hormone can be administered orally, nasally, and intravenously. This medication promotes the release of factor VIII and vWf from storage. Cryoprecipitate may be used; however, because of the risk of possible donor viruses, other modalities are considered safer. Treatment with plasma products such as factor VIII and vWf is an acceptable option for this client. Because of the repeated exposure to donor blood products and possible viruses, this is not the initial treatment of choice. Although the administration of this prostaglandin is known to promote contraction of the uterus during postpartum hemorrhage, it is not effective for the client who presents with a bleeding disorder. C

The nurse is developing a dietary teaching plan for a patient on a vegetarian diet. The nurse should provide the client with which examples of protein-containing foods? Select all that apply. A. Dried beans B. Seeds C. Peanut butter D. Bagel E. Peas

Dried beans, seeds, peanut butter, and peas provide protein. A bagel is an example of a whole grain food, not protein. A, B, C, E

The nurse instructs a pregnant client to wear a supportive bra. How would this intervention benefit the client? A. Arching of the back as compensation B. Improvement in respiratory status C. Reduction of pressure on the ulnar nerve D. Alleviation of disturbed body image

During pregnancy, increased breast size and weight results in increased pressure on the ulnar nerve. Hence, the nurse instructs the client to wear a supportive bra. Arching of the back is compensated for by wearing supportive shoes. Wearing a supportive bra does not improve respiratory status, because respiratory changes in pregnancy are generally due to increases in fundal height. A supportive bra is not intended alleviate disturbed body image; it is meant to relieve pressure on the ulnar nerve. C

The nurse is caring for a client with a singleton pregnancy. What is the expected total weight gain for the client during the first trimester? A. 2 to 3 kg B. 1 to 2 kg C. 1.5 to 2 kg D. 0.5 to 1 kg

During the first and second trimesters, growth takes place primarily in maternal tissues. During the third trimester, growth occurs primarily in fetal tissues. During the first trimester of a singleton pregnancy, the average total weight gain is only 1 to 2 kg. Greater-than-expected weight gain during pregnancy, such as 2 to 3 kg, can occur for various reasons, including multiple gestation, edema, gestational hypertension, and overeating. Weight gain of 1.5 to 2 kg or 0.5 to 1 kg may occur due to improper diet, which may lead to growth problems in the fetus. B

A client in the first trimester of pregnancy tells the nurse, "I urinate frequently and am not able to hold urine even for a short time." What would the nurse suggest to ease the client's discomfort? A. "Eat dry carbohydrates." B. "Perform Kegel exercises." C. "Apply local heat or ice." D. "Get back rubs regularly."

During the first trimester of pregnancy, clients may have various discomforts such as urgent urination. Kegel exercises help strengthen the pelvic floor muscles and thus are helpful decreasing urinary urgency. Dry carbohydrates are included in the diet to suppress vomiting in pregnant women, but they have no effect on urinary urgency. Headache is also a common discomfort in a pregnant client. Massage and hot and cold application help relieve this pain. Backache during pregnancy can be eased by giving back rubs to a pregnant client. B

The nurse is caring for an underweight patient with singleton pregnancy. After the first trimester, the nurse observes that the patient's weight gain is approximately 0.3 kg per week. Which risk is the fetus exposed to? A. Hypoglycemia B. Hypocalcemia C. Congenital malformations D. Intrauterine growth restriction (IUGR)

During the first trimester of singleton pregnancy, the average total weight gain is only 1 to 2 kg. Thereafter, the recommended weight gain increases to approximately 0.5 kg per week for an underweight patient. Therefore, the fetus of a pregnant patient who has gained less than 0.5 kg per week may be at risk for intrauterine growth restriction (IUGR). The fetus is at risk for hypoglycemia if the mother is hypoglycemic. The fetus is at risk for hypocalcemia if the mother has an inadequate amount of calcium in her daily diet during pregnancy. The fetus is at risk for congenital disorders if the mother has impaired nutrition during pregnancy. D

A 22-year-old woman pregnant with a single fetus had a preconception body mass index (BMI) of 24. When she was seen in the clinic at 14 weeks of gestation, she had gained 1.8 kg (4 lbs) since conception. How would the nurse interpret this finding? A. This weight gain indicates possible gestational hypertension. B. This weight gain indicates that the woman's infant is at risk for intrauterine growth restriction (IUGR). C. This weight gain cannot be evaluated until the woman has been observed for several more weeks. D. The woman's weight gain is appropriate for this stage of pregnancy.

During the first trimester, the average total weight gain is only 1 to 2.5 kg. The desirable weight gain during pregnancy varies among women. Weight gain should take place throughout the pregnancy. The optimal rate depends on the stage of the pregnancy. The primary factor to consider in making a weight gain recommendation is the appropriateness of the prepregnancy weight for the woman's height. A commonly used method of evaluating the appropriateness of weight for height is the BMI. This woman's BMI is within the normal range, and she has gained the appropriate amount of weight for her size at this point in her pregnancy. Although the statements in A through C are accurate, they do not apply to this client. D

A primary health care provider orders an ultrasound for a pregnant client before attempting external cephalic version (ECV). Upon assessing the client's ultrasound report, the nurse suspects that the primary health care provider will not attempt ECV. Which findings support the nurse's expectation? Select all that apply. A. The client has a nuchal cord. B. The client is Rh negative. C. The client has oligohydramnios. D. The fetal heart rate is 120 beats per minute. E. The client has uterine anomalies

ECV is performed to change the fetus from a breech to a vertex presentation by applying pressure on the abdomen. ECV is contraindicated in certain conditions, including the presence of a nuchal cord, oligohydramnios, and uterine anomalies. ECV should be avoided if the ultrasound shows any of the complications mentioned. ECV is not contraindicated in Rh-negative client. Patients with an Rh-negative blood group are administered Rh immunoglobulin before performing ECV. A fetal heart rate of 120 beats per minute is considered normal, and ECV is not contraindicated in this condition. A, C, E

A postpartum client who underwent a cesarean delivery complains of minor perineal and rectal pain. Further assessment shows the presence of peritoneal hematomas. Which primary nursing interventions should be performed after surgically removing the hematomas and minimizing the bleeding? Select all that apply. A. Prepare for hysterectomy. B. Administer pain relief medication. C. Initiate fluid replacement therapy. D. Monitor serum hemoglobin levels. E. Monitor the amount of blood loss.

Early manifestations of retroperitoneal hematoma include shock, which is caused by excessive bleeding. After the surgical removal of the hematomas, the priority nursing intervention is to initiate fluid replacement therapy to treat the shock. Along with fluid replacement, the nurse should monitor the client's hemoglobin levels until they are restored to normal values. Monitoring blood loss helps the nurse assess the client's condition. Hysterectomy is performed when there is excessive bleeding that is not responsive to conservative measures. Retroperitoneal hematoma is associated with minimal pain, so administering pain relief medication is not a priority nursing intervention. C, D, E

A pregnant woman experiencing nausea and vomiting should: A. Drink a glass of water with a fat-free carbohydrate before getting out of bed in the morning. B. Eat small, frequent meals (every 2 to 3 hours). C. Increase her intake of high-fat foods to keep the stomach full and coated. D. Limit fluid intake throughout the day.

Eating small, frequent meals is a correct suggestion for a pregnant woman experiencing nausea and vomiting. She should avoid consuming fluids early in the day or when nauseated, but should compensate by drinking fluids at other times. She should also reduce her intake of fried foods and other fatty foods. B

Which findings could be considered to be a barrier to a pregnant woman seeking prenatal care? (Select all that apply.) A. Patient would prefer to be cared for by a midwife instead of a physician. B. Economic cost of health care. C. Patient's cultural beliefs do not include prenatal care as being valued. D. Patient speaks several languages. E. Patient had a bad experience the last time she went to a doctor for care.

Economic factors can delay the onset of health care treatment. A patient's cultural beliefs and values may be a barrier to seeking prenatal care if her culture does not perceive any inherent value in prenatal care. If the patient had a bad prior experience with a health care provider, it may be a barrier to seeking future care. The fact that this patient is multilingual does not necessarily represent a barrier to seeking prenatal care. Although the patient may prefer to be cared for by a midwife, this fact cannot be considered a barrier to seeking prenatal care because it demonstrates a patient's choice. B, C, E

The postpartum client who delivered a day ago reports, "I feel tired very often and experience pain in my lower abdomen." Upon further observation, the nurse finds that the client also has profuse foul-smelling vaginal discharge and an increased pulse. Which medication would be added to the client's prescription? A. Warfarin (Coumadin) B. Clindamycin (Cleocin) C. Misoprostol (Cytotec) D. Ergonovine (Ergotrate)

Endometrial infection is characterized by tiredness and lower abdominal pain, profuse foul-smelling discharge, and increased pulse. Clindamycin (Cleocin) is an antibiotic used to treat endometrial infections. Warfarin (Coumadin) is prescribed to postpartum clients with thromboembolic disorders. Misoprostol (Cytotec) is prescribed to clients with excessive bleeding caused by uterine atony. Ergonovine (Ergotrate) is prescribed to treat subinvolution of the uterus. B

If a pregnant client suspects signs and symptoms of preterm labor, which conditions would lead the client to go to hospital immediately? Select all that apply. A. Nausea and vomiting B. Upper back pain C. Fluid leakage from vagina D. Presence of vaginal bleeding E. Contractions every 10 minutes

Fluid leakage from the vagina indicates rupture of the amniotic membranes. The client should seek immediate medical attention, because ruptured amniotic membranes can compromise fetal health. Presence of vaginal bleeding may indicate onset of labor or placental hemorrhage, which may compromise fetal perfusion. Therefore the client should go to the hospital immediately. Uterine contractions (UCs) after every 10 minutes indicate active labor and the client should go to the hospital immediately. Nausea and vomiting and upper back pain do not indicate labor. The client need not seek immediate medical attention for these conditions. C, D, E

The nurse is assessing a 25-year-old pregnant client and learns the client is lactose intolerant and avoids consuming any dairy foods. Upon reviewing the client's daily diet chart, the nurse sees that the client eats four pieces of French toast and three cups of cooked dried beans almost every day. The client does not consume any other calcium-rich food in the diet. What does the nurse interpret about the client? A. The client consumes sufficient calcium. B. The client requires additional calcium. C. The client requires calcium from different sources. D. The client is consuming too much calcium.

Four pieces of French toast and three cups of dried beans is equivalent to about two cups of milk in terms of the calcium content. Each cup of milk contains approximately 300 mg of calcium, so the client is getting about 600 mg of calcium daily. The recommended intake of calcium for pregnant women over the age of 19 is 1000 mg/day so the client is falling short of the daily calcium requirement. The nurse should suggest that the client add another 400 mg of calcium every day. Because the total calcium requirement is 100 mg and the client is already getting some calcium, the nurse would not suggest that the client consume an additional 1000 mg of calcium. C

After massaging the boggy fundus of a client who delivered a large baby after a prolonged labor with a forceps-assisted birth, the nurse is unable to obtain a firm fundus. What nursing action is indicated at this time? A. Increase the rate of the intravenous infusion. B. Massage the fundus while another nurse notifies the PCP. C. Change the peripad, replacing it with a double pad. D. Administer a half-dose of a uterine contracting medication.

Fragments of the placenta can remain in the uterus after spontaneous separation of the placenta during the third stage of labor. In this case, the woman will have excessive bleeding and the uterus will feel boggy (soft) because of uterine atony. Ultrasonography can be used to detect placental fragments. The client should not be left alone because of the boggy uterus, which can lead to hemorrhage. There is an absolute need for further medical intervention. B

The nurse is assessing a pregnant client and finds that the client has had spinal surgery. What does the nurse interpret from the assessment? A. The client may have higher chances of preterm delivery. B. Cesarean birth should be recommended for the client. C. Epidural anesthesia is contraindicated in the client. D. The client may have right lower quadrant pain during pregnancy.

From the assessment the nurse determines that the client has a history of spinal surgery, and epidural anesthesia can lead to severe complications in such clients. A history of spinal surgery does not cause preterm delivery or cesarean birth. If the client has had uterine surgery or extensive repair of the pelvic floor, then cesarean birth would be recommended for the client. Unlike appendicitis, spinal surgery does not cause right lower quadrant pain during pregnancy. C

With regard to protein in the diet of pregnant women, what should the nurse be aware of? A. Many protein-rich foods are also good sources of calcium, iron, and B vitamins. B. Many women need to increase their protein intake during pregnancy. C. As with carbohydrates and fat, no specific recommendations exist for the amount of protein in the diet. D. High-protein supplements can be used without risk by women on macrobiotic diet.

Good protein sources such as meat, milk, eggs, and cheese also have a lot of calcium and iron. Most women already eat a high-protein diet and do not need to increase their intake. Protein is sufficiently important that specific servings of meat and dairy are recommended. High-protein supplements are not recommended, because they have been associated with an increased incidence of preterm births. A

With regard to protein in the diet of pregnant women, nurses should be aware that: A. Many protein-rich foods are also good sources of calcium, iron, and B vitamins. B. Many women need to increase their protein intake during pregnancy. C. As with carbohydrates and fat, no specific recommendations exist for the amount of protein in the diet. D. High-protein supplements can be used without risk by women on macrobiotic diets.

Good protein sources such as meat, milk, eggs, and cheese have a lot of calcium and iron. Most women already eat a high-protein diet and do not need to increase their intake. Protein is sufficiently important that specific servings of meat and dairy are recommended. High-protein supplements are not recommended because they have been associated with an increased incidence of preterm births. A

A woman diagnosed with marginal placenta previa gave birth vaginally 15 minutes ago. At the present time, she is at the greatest risk for what problem? A. Hemorrhage B. Infection C. Urinary retention D. Thrombophlebitis

Hemorrhage is the most immediate risk because the lower uterine segment has limited ability to contract to reduce blood loss. Infection is a risk because of the location of the placental attachment site; however, it is not a priority concern at this time. Placenta previa poses no greater risk for urinary retention than with a normally implanted placenta. There is no greater risk for thrombophlebitis than with a normally implanted placenta. A

The quantitative human chorionic gonadotropin (β-hCG) levels are high in a client who is on methotrexate therapy for dissolving abdominal pregnancy. Which instruction does the nurse give to this client? A. "Avoid sexual activity." B. "Avoid becoming pregnant again." C. "Avoid feeling sad and low." D. "Take folic acid without fail."

High β-hCG levels indicate that the abdominal pregnancy is not yet dissolved. Therefore the nurse advises the client to avoid sexual activity until the β-hCG levels drop and the pregnancy is dissolved completely. If the client engages in vaginal intercourse, the pelvic pressure may rupture the mass and cause pain. Abdominal pregnancy increases the chances of infertility or recurrent ectopic pregnancy in clients. However, the nurse need not instruct the client to avoid further pregnancy, because it may increase the feelings of sadness and guilt in the client. The nurse encourages the client to share feelings of guilt or sadness related to pregnancy loss. Folic acid is contraindicated with methotrexate therapy, because it may exacerbate ectopic rupture. A

A pregnant client has severe and persistent vomiting. The client has lost weight, is dehydrated, and has electrolyte abnormalities. Which condition does the nurse suspect that the client has? A. Tetany B. Glossitis C. Hypocalcemia D. Hyperemesis gravidarum

Hyperemesis gravidarum is a condition associated with severe and persistent vomiting causing weight loss, dehydration, and electrolyte abnormalities. Tetany is a condition caused by vitamin D deficiency and is characterized by muscle cramps, spasms, or tremors. Glossitis is characterized by an inflamed red tongue; it does not lead to severe and persistent vomiting but may lead to weight loss. Hypocalcemia may lead to retardation of bone development in the infant. D

A postpartum woman with hypovolemic shock is given an intravenous infusion of crystalloids. Upon assessing the reports of the blood tests done after the infusion, the nurse finds that the client has decreased clotting factors and platelet count. Which intervention would help in restoring the fluid electrolyte balance? A. Administration of fresh frozen plasma B. Administration of packed red blood cells C. Administration of normal saline solution D. Administration of lactated Ringer's solution

Hypovolemic shock is initially treated by restoring blood volume and eliminating the underlying cause of the shock. The client is initially treated by administering crystalloids in order to perform fluid resuscitation. Because the fresh frozen plasma is rich in clotting factors and platelets, administration of fresh frozen plasma restores the normal levels of platelets and clotting factors in the client. Packed red blood cells are administered if the client is actively bleeding. Normal saline solution and lactated Ringer's solution are both crystalline solutions that are administered to restore the circulating blood volume in the condition of hypovolemic shock. However, these solutions are not helpful in restoring normal levels of platelets and clotting factors. A

A pregnant woman reports that she is still playing tennis at 32 weeks of gestation. The nurse would be most concerned regarding what this woman consumes during and after tennis matches. Which is the most important? A. Several glasses of fluid B. Extra protein sources, such as peanut butter C. Salty foods to replace lost sodium D. Easily digested sources of carbohydrate

If no medical or obstetric problems contraindicate physical activity, pregnant women should get 30 minutes of moderate physical exercise daily. Liberal amounts of fluid should be consumed before, during, and after exercise, because dehydration can trigger premature labor. Also the woman's calorie and carbohydrate intake should be sufficient to meet the increased needs of pregnancy and the demands of exercise. All pregnant women should consume the necessary amount of protein in their diet, regardless of level of activity. Many pregnant women of this gestation tend to retain fluid, possibly contributing to hypertension and swelling. An adequate fluid intake prior to and after exercise should be sufficient without trying to replace sodium. A

The nurse is assessing a client with postpartum hemorrhage (PPH). During the physical assessment, the nurse finds that there are deep lacerations in the cervix. Which observation allows the nurse to conclude that the PPH is due to cervical lacerations? A. Dark red blood B. Bright red blood C. Clots in the blood D. Foul-smelling blood

If the color of blood is bright red, it indicates that hemorrhage has occurred due to deep lacerations of the cervix. Foul-smelling blood during the postpartum period indicates infection. Bleeding due to varices or superficial lacerations of the birth canal is dark red in color. Clots in the blood characterize the PPH caused by disseminated intravascular coagulation (DIC). B

The nurse is teaching a group of pregnant clients about preterm labor and the actions to take if the signs and symptoms of preterm labor develop. Which patient statement indicates the need for further teaching? A. "I will empty my bladder immediately." B. "I will drink 2 to 3 glasses of water or juice." C. "I will lie in the supine position for 1 hour." D. "I will go to hospital if symptoms continue."

If there are signs and symptoms of preterm labor, the client should lie down on her side for 1 hour, because it helps improve placental and fetal circulation. The client should empty her bladder immediately, because a full bladder may sometimes irritate the uterus. Dehydration may also irritate the uterus. Therefore, the client should drink 2 to 3 glasses of water or juices. The patient should go to the hospital if the symptoms of preterm labor do not subside. C

The nurse is caring for a pregnant adult Puerto Rican client. Which food does the nurse instruct the client to consume on a daily basis to prevent calcium imbalance? A. Milk B. Collards C. Yogurt D. Cheese

In some cultures, such as the Puerto Rican, it is uncommon for adults to drink milk. These cultures may use milk only as an additive in coffee. Pregnant women in the Puerto Rican culture may need to consume nondairy sources of calcium. Collards are a good nondairy source of calcium. Therefore, the nurse can instruct the client to consume collards instead of dairy sources such as milk, yogurt, or cheese. B

The nurse is caring for an adolescent client who is pregnant and has conflict with the family members regarding the childbirth and future career plans. What is the best nursing intervention to help the client reduce conflict within her family? A. Refer the client to a support group. B. Refer the client to parenting classes. C. Encourage the client to verbalize her fears. D. Provide an opportunity to discuss the client's personal feelings.

Interrupted family processes due to adolescent pregnancy should be addressed carefully. Referring the client to a support group helps her develop effective problem-solving skills and reduce conflicts within her family. Referring the client to parenting classes may teach her about basic newborn care, but may not be helpful in counseling about future career options. A nursing intervention aimed at verbalizing fears helps reduce anxiety in the client. Providing an opportunity to discuss personal feelings may be an appropriate intervention for pregnant adolescents who have a disturbed body image. A

While examining the breasts of a pregnant client, the nurse observes that the client has inverted nipples. What does the nurse interpret from this finding? A. The client had a breast reduction surgery in the past. B. The client has to apply iodine tincture on the nipples. C. The client has normal breast changes during pregnancy. D. The client's baby will probably have difficulty latching.

Inverted nipples affect the infant's ability to latch onto the breast, so the infant will have difficulty sucking the nipple during breastfeeding. Scars on the breast are an indication of previous breast reduction surgery. Tinctures should not be applied to the breast because they remove protective oils that keep nipples supple. This may cause cracks on the nipple during early lactation. A normal nipple will stand erect when pressure is applied. Therefore, presence of inverted nipples is not a normal finding during pregnancy. D

Which minerals and vitamins are usually recommended to supplement a pregnant woman's diet? A. Fat-soluble vitamins A and D B. Water-soluble vitamins C and B6 C. Iron and folate D. Calcium and zinc

Iron generally should be supplemented, and folic acid supplements often are needed because folate is so important. Fat-soluble vitamins should be supplemented as a medical prescription, as vitamin D might be for lactose-intolerant women. Water-soluble vitamin C sometimes is consumed in excess naturally; vitamin B6 is prescribed only if the woman has a very poor diet. Zinc is sometimes supplemented; most women get enough calcium. C

The nurse is caring for a pregnant client who is taking iron supplements. What is the risk of iron supplementation for pregnant women? A. Tetany B. Anemia C. Diabetes D. Constipation

Iron supplements are prescribed to increase the client's folate levels. Constipation is generally observed in clients who consume iron supplements, so clients are advised to eat a diet high in fiber to relieve constipation. Tetany is caused by vitamin D deficiency. Anemia may be caused by folic acid deficiency. Diabetes is caused by low insulin levels. D

Which foods does the nurse exclude from the pregnant client's diet plan to ensure good health? Select all that apply. A. Meats B. Butter C. Yogurt D. Beef fat E. Stick margarine

It is advisable to include oils rather than solid fats in the diet plan of a pregnant client. Solid fats are fats that are solid at room temperature and cause increased body weight. This may result in greater BMI and obesity. Therefore, solid fats such as butter, beef fat, and stick margarine should be avoided in the diet plan of a pregnant client. Meat is rich in folate, and yogurt is rich in vitamin D. Therefore, these products are recommended for the diet of a pregnant client. B, D, E

A student nurse is teaching a group of pregnant women about sibling adaptation. Which statement by one of the clients indicates a need for additional teaching? A. "Show the child how to touch the baby." B. "Exclude the child during infant feeding times." C. "Don't force interactions between the child and the baby." D. "Help the child to have realistic expectations about the baby."

It is very important to help siblings adapt to their mother's pregnancy and accept the arrival of a new baby into the family. The child shouldn't be excluded during infant feeding times, because this may cause a feeling of separation. Mothers should be encouraged to teach their children how and where to touch the baby. This encourages the child to get attached to the baby. The child should be left free while interacting with the neonate, but interactions should not be forced. The child should be encouraged to have realistic expectations about what babies are like. B

A pregnant client in the third trimester reports urinary frequency. What is the best nursing intervention for reducing frequency of urination in the client? A. Advise the client to limit her fluid intake. B. Encourage the client to wear a perineal pad. C. Advise the client to empty her bladder regularly. D. Encourage the client to perform Kegel exercises.

Kegel exercises involve deliberate contraction and relaxation of the pubococcygeus muscle. Practicing Kegel (pelvic muscle) exercises during pregnancy results in fewer complaints of urinary incontinence. Limiting the client's fluid intake may cause dehydration. Fluid intake should be limited only during the night. Wearing a perineal pad reduces discomfort associated with urinary frequency but does not affect the frequency of urination. Regularly emptying the bladder does not reduce urinary frequency. D

The nurse is caring for a pregnant client who is diagnosed with a urinary tract infection (UTI). Which symptoms of a UTI does the nurse expect to find in the client? Select all that apply. A. Dysuria B. Dribbling C. Hematuria D. Urinary frequency E. Odor of vaginal discharge

Lower urinary tract infections occur because of physiologic changes during pregnancy. Dysuria, or painful urination, may be caused by acidic pH of the urine due to infection. Urine may dribble, because the infection may cause impaired function of the urethral sphincter. The urine may contain red blood cells, leading to hematuria. Impaired urethral sphincter and irritation of the bladder mucosa may lead to increased urinary frequency. A vaginal infection would produce a foul odor in vaginal discharge. A, B, C, D

The nurse administers magnesium sulfate (Epsom salts) to stop labor in a pregnant client. Which symptoms should the nurse monitor to ensure the client's safety? A. Swollen legs B. Respiratory rate C. Eating patterns D. Maternal chills

Magnesium sulfate (Epsom salts) is administered to a pregnant client to stop labor. Magnesium sulfate (Epsom salts) causes respiratory depression as a toxic effect. Therefore, the nurse should monitor the respiratory rate of the client. Swollen legs or edema is acommon observation during labor, which is caused by increased abdominal contents. Edema is unrelated to magnesium sulfate. Magnesium sulfate (Epsom salts) does not alter a client's eating habits. Maternal chills are observed in clients with membrane rupture and are unrelated to magnesium sulfate (Epsom salts). B

A pregnant client has preterm labor. Which medication would be helpful in preventing preterm labor in the client? A. Oxytocin (Pitocin) B. Methotrexate (MTX) C. Misoprostol (Cytotec) D. Magnesium sulfate (Sulfamag)

Magnesium sulfate (Sulfamag) is a tocolytic agent used to slow down uterine contractions. Oxytocin (Pitocin) is used to prevent uterine hemorrhage. MTX is used for the treatment of ectopic pregnancy. Misoprostol (Cytotec) is used to expel the products of conception completely after a miscarriage. D

The nurse is caring for a pregnant client who is administered magnesium sulfate to prevent preterm labor. Which parameters should the nurse assess in the patient to determine drug toxicity? Select all that apply. A. Fluid intake B. Respiratory status C. Body temperature D. Level of consciousness E. Deep tendon reflexes

Magnesium sulfate, when used as a tocolytic agent, depresses the central nervous system (CNS). The CNS depressive effect would be enhanced if the drug reaches toxic levels. CNS activity can be determined by assessing the respiratory status, level of consciousness, and deep tendon reflexes. A low respiratory rate, decreased level of consciousness, and slow reflexes indicate magnesium sulfate toxicity. Fluid intake and body temperature are not affected by CNS depression. B, D, E

A nurse is making rounds on a client who recently delivered via the vaginal route, and suspects that the client is having excessive postpartum bleeding. Which intervention would be the priority action taken by the nurse at this time? A. Call the physician B. Massage the uterine fundus C. Increase the rate of intravenous fluids C. Monitor pad count and perform catherization

Massaging of the uterine fundus would be a priority action to help expel clots and stimulate uterine contractions to constrict blood flow. Although the physician may well have to be called, the initial priority action would be for the nurse to massage the uterine fundus. Although the rate of intravenous fluids may have to be increased, this is not the priority action at this time. Monitoring of pad count and lochia flow is needed but it is not the priority action at this time. Additionally, catherization may be needed if bladder distension is noted but, again, it is not the priority action at this time. B

During rounds, a nurse suspects that a patient who has recently delivered via vaginal route is having excessive postpartum bleeding. Which intervention would be the priority action taken by the nurse at this time? A. Call the physician. B. Massage the uterine fundus. C. Increase the rate of intravenous fluids. D. Monitor pad count and perform catheterization.

Massaging of the uterine fundus would be a priority action to help expel clots and stimulate uterine contractions to constrict blood flow. The other actions described, as well as catheterization (if bladder distention is noted) and lochia flow monitoring, may be needed, but none of them is the priority action required at this time. B

Which postpartum infection is most often contracted by first-time mothers who are breastfeeding? A. Endometritis B. Wound infections C. Mastitis D. Urinary tract infections (UTIs)

Mastitis is infection in a breast, usually confined to a milk duct. Most women who get it are first-timers who are breastfeeding. Endometritis is the most common postpartum infection. Its incidence is higher after a cesarean birth, not in first-time mothers. Wound infections are also a common postpartum complication. Sites of infection include both a cesarean incision and the episiotomy or repaired laceration. The gravidity of the mother and her feeding choice are not factors in the development of a wound infection. UTIs occur in 2% to 4% of all postpartum women. Risk factors include catheterizations, frequent vaginal exams, and epidural anesthesia. B

Which postpartum infection is most often contracted by first-time mothers who are breastfeeding? A. Endometritis B. Wound infections C. Mastitis D. Urinary tract infections (UTIs)

Mastitis is infection in a breast, usually confined to a milk duct. Most women who suffer this are first-timers who are breastfeeding. Endometritis is the most common postpartum infection. Incidence is higher after a cesarean birth and not limited to first-time mothers. Wound infections are also a common postpartum complication. Sites of infection include both a cesarean incision, and the episiotomy or repaired laceration. The gravidity of the mother and her feeding choice are not factors in the development of a wound infection. UTIs occur in 2% to 4% of all postpartum women. Risk factors include catheterizations, frequent vaginal exams, and epidural anesthesia. C

The nurse advises the woman who wants to have a nurse-midwife provide obstetric care that: A. She will have to give birth at home. B. She must see an obstetrician as well as the midwife during pregnancy. C. She will not be able to have epidural analgesia for labor pain. D. She must be having a low-risk pregnancy.

Midwives usually see low-risk obstetric clients. Nurse-midwives must refer clients to physicians for complications. Most nurse-midwife births are managed in hospitals or birth centers; a few may be managed in the home. Nurse-midwives may practice with physicians or independently with an arrangement for physician backup. They must refer clients to physicians for complications, but patients are not required to see an obstetrician otherwise. Care in a midwifery model is noninterventional, and the woman and family usually are encouraged to be active participants in the care; this does not imply that medications for pain control are prohibited. D

A client reports mild vaginal bleeding, pain, and cramping in her lower abdomen at 6 weeks of gestation. Upon performing a pelvic examination, the nurse finds that the client's cervical os is closed. What is the priority intervention in this condition? A. Administer intravenous fluids to the client. B. Administer methylcarboprost tromethamine (Hemabate) to the client. C. Determine the client's human chorionic gonadotropin and progesterone levels. D. Prompt termination of pregnancy in the patient by dilation and curettage method.

Mild vaginal bleeding, pain in the lower abdomen, and mild uterine cramps with a closed cervical os are the symptoms of threatened miscarriage. Therefore, the nurse should determine the human chorionic gonadotropin and progesterone levels in the client to find whether the fetus is alive in the uterus. The nurse should administer intravenous fluids if the client has severe bleeding. The nurse should administer methylcarboprost tromethamine (Hemabate) to prevent excessive bleeding after miscarriage. In this case, the client does not have a complete miscarriage or excessive bleeding; therefore, the nurse would not administer this medication to the client. Dilation and curettage is a surgical method to terminate a pregnancy. It is performed on a client with inevitable miscarriage, but not threatened miscarriage. C

The most appropriate statement that the nurse can make to bereaved parents is: A. "You have an angel in heaven." B. "I understand how you must feel." C. "You're young and can have other children." D. "I'm sorry."

One of the most important goals of the nurse is to validate the experience and feelings of the parents by encouraging them to tell their stories and listening with care. At the very least, the nurse should acknowledge the loss with a simple but sincere comment, such as, "I'm sorry." The initial impulse may be to reduce one's sense of helplessness and to say or do something that you think will reduce their pain. Although such a response may seem supportive at the time, it can stifle the further expression of emotion. The nurse should resist the temptation to give advice or to use clichés when offering support to the bereaved. The statement in C is not a therapeutic response for the nurse to make. D

In understanding and guiding a woman through her acceptance of pregnancy, a maternity nurse should be aware that: A. Nonacceptance of the pregnancy very often equates to rejection of the child. B. Mood swings are most likely the result of worries about finances and a changed lifestyle, as well as profound hormonal changes. C. Ambivalent feelings during pregnancy are usually seen only in emotionally immature or very young mothers. D. Conflicts such as not wanting to be pregnant or childrearing and career-related decisions need not be addressed during pregnancy because they will resolve themselves naturally after birth.

Mood swings are natural and are likely to affect every woman to some degree. A woman may dislike being pregnant, refuse to accept it, and still love and accept the child. Ambivalent feelings about pregnancy are normal for mature or immature women, young or older. Conflicts about desire to perform childrearing and career-related concerns, however, need to be resolved; the baby's arrival ends the pregnancy but not all the issues. B

A client who is in the first trimester of pregnancy reports frequent nausea. Which nursing interventions would help relieve the symptoms of nausea in this client? Select all that apply. A. Assessing if the client is well hydrated B. Assessing the client's weight gain pattern during pregnancy C. Reviewing the food frequency approach during pregnancy D. Reviewing measures already taken for the prevention of morning sickness E. Discussing food cravings that may occur during pregnancy

Nausea may occur due to morning sickness during the first trimester of pregnancy. To relieve this condition in the client, the nurse should assess the reasons behind it. Therefore, the nurse is required to assess the client's state of hydration and pattern of weight gain during pregnancy. The nurse should also find out what measures the client has taken to relieve morning sickness. Then, the nurse can treat the condition of nausea in the client. Reviewing the food frequency approach and discussing food cravings that may occur during pregnancy would help prevent nutritional imbalance, because it is possible that the client is avoiding food for fear of weight gain. A, B, D

The nurse is caring for a postpartum client. On assessment, the nurse finds that the client's neonate has neural tube defects (NTD). Which intervention would be beneficial for the client? A. Daily calcium supplement of 0.4 mg B. Daily calcium supplement of 0.8 mg C. Daily folic acid supplement of 1.0 mg D. Daily folic acid supplement of 0.4 mg

Neural tube defects (NTDs) are more common in infants born of mothers with low folic acid intake during pregnancy. A patient who has had a pregnancy involving a child with NTD should take 0.4 mg of folic acid daily, even if not planning for another pregnancy. Neither 0.4 mg nor 0.8 mg of calcium will help prevent neural tube defects. A dose of 1.0 mg of folic acid is too high and may decrease the absorption of vitamin B12 in the client. D

Which food would be a common protein source for a Mexican client who is pregnant? A. Veal B. Mussels C. Dal (lentils) D. Chorizo (sausage)

Our cultural and ethnic backgrounds influence our diets. A Mexican client may get her protein from chorizo if she is eating traditional Mexican meals during her pregnancy. Mussels, typically preferred by Native Americans, and veal, which may be preferred by Italians, are also rich in protein. Dal (lentils) may be a preferred source protein for Indians. D

Which of the following is the most common kind of placental adherence seen in pregnant women? A. Accreta B. Placenta previa C. Percreta D. Increta

Placenta accreta is the most common kind of placental adherence seen in pregnant women and is characterized by slight penetration of myometrium. In placenta previa, the placenta does not embed correctly and results in what is known as a low-lying placenta. It can be marginal, partial, or complete in how it covers the cervical os, and it increases the patient's risk for painless vaginal bleeding during the pregnancy and/or delivery process. Placenta percreta leads to perforation of the uterus and is the most serious and invasive of all types of accrete. Placenta increta leads to deep penetration of the myometrium. A

The nurse assesses that a client in the third trimester of pregnancy is having severe vomiting, headaches, epigastric pain, muscular irritability, and swelling of the fingers. What risk does the nurse expect in the client? A. Preeclampsia B. Hyperemesis C. Fetal jeopardy D. Placenta previa

Preeclampsia is a hypertensive condition observed in pregnant women, characterized by swollen face and fingers, muscular irritability, severe headache, and vomiting. Hyperemesis gravidarum only involves increased vomiting, even after the first trimester of pregnancy. Fetal jeopardy or fetal death is indicated by absence of fetal movements and fetal heart rate. Placenta previa is indicated by vaginal bleeding and severe abdominal pain. A

The nurse is teaching a pregnant client about body mechanics to decrease discomfort related to the lumbar curve of pregnancy. Which statement made by the client indicates the need for additional teaching? A. "I should avoid platform shoes and high heels." B. "I should use a pillow in the car to support my lower back area." C. "I should change positions often if I have to stand for a long time." D. "I should adjust my car seat such that my knees are lower than my hips."

Pregnant clients should be taught about posture and body mechanics to restrict the lumbar curve. The car seat should be moved forward in such a way that the knees are higher than the hips. Pregnant women should avoid wearing high-heeled or platform shoes because they cause anterior tilting of the pelvis and therefore, increase the lumbar curve. A pillow should be used in the car to support the lower back area. Position should be changed often during prolonged standing to relieve weight-bearing stresses on the client's joints. D

In caring for the woman with disseminated intravascular coagulation (DIC), what order should the nurse anticipate? A. Administration of blood B. Preparation of the woman for invasive hemodynamic monitoring C. Restriction of intravascular fluids D. Administration of steroids

Primary medical management in all cases of DIC involves correction of the underlying cause, volume replacement (not volume restriction), blood component therapy, optimization of oxygenation and perfusion status, and continued reassessment of laboratory parameters. Central monitoring would not be ordered initially in a woman with DIC because it could contribute to more areas of bleeding. Steroids are not indicated for the management of DIC. A

What are probable indicators of pregnancy? Select all that apply. A. Ballottement B. Urinary frequency C. Nausea and vomiting D. Uterine enlargement E. Braxton Hicks contractions

Probable indicators of pregnancy include ballottement, uterine enlargement, Braxton Hicks contractions, placental souffle, and a positive pregnancy test. Probable indicators combined with presumptive signs strongly suggest pregnancy. Urinary frequency, nausea, and vomiting are presumptive indicators of pregnancy, and these signs alone are not reliable for the diagnosis of pregnancy. A, D, E

While assessing a postpartum client the nurse finds that the client has excessive foul smelling lochia. What medication would be helpful in treating the condition? A. A broad-spectrum antibiotic B. A diuretic to induce urination C. Intravenous oxytocin agents D. Intravenous fluids

Profuse lochia with a foul smell indicates that the client has a bacterial infection, such as endometritis. Therefore, the nurse anticipates that the PHP will prescribe a broad-spectrum antibiotic. Diuretics are prescribed when the client shows urinary retention, characterized by decreased urinary output. Intravenous oxytocin agents are prescribedto induce uterine contractions in case of uterine inversion. The nurse should ensure adequate fluid intake; however, administration of intravenous fluids is likely not necessary. A

Which condition in the client may lead to uterine atony? Select all that apply. A. Primigravidity B. Prolonged labor C. Polyhydramnios D. Precipitous birth E. Fetal macrosomia

Prolonged labor causes the uterus to expand for a longer period of time, which may result in uterine atony. Polyhydramnios causes the uterus to overstretch, leading to uterine atony. Excessive stretching of the uterine muscles may occur due to the presence of a large fetus. It is called fetal macrosomia. Primigravidity does not cause uterine atony, because it does not affect the uterine muscles. Precipitous birth, or rapid birth, does not cause uterine atony, but may lead to vaginal and cervical lacerations. B, C, E

Signs of a threatened abortion (miscarriage) are noted in a woman at 8 weeks of gestation. What is an appropriate management approach? A. Prepare the woman for a dilation and curettage (D&C). B. Place the woman on bed rest for at least 1 week and reevaluate. C. Prepare the woman for an ultrasound and blood work. D. Comfort the woman by telling her that if she loses this baby, she may attempt to get pregnant again in 1 month.

Repetitive transvaginal ultrasounds and measurement of human chorionic gonadotropin (hCG) and progesterone levels may be performed to determine if the fetus is alive and within the uterus. If the pregnancy is lost, the woman should be guided through the grieving process. D&C is not considered until signs of the progress to an inevitable abortion are noted or the contents are expelled and incomplete. Bed rest is recommended for 48 hours initially. Telling the woman that she can get pregnant again soon is not a therapeutic response because it discounts the importance of this pregnancy. C

Signs of a threatened abortion (miscarriage) are noted in a woman at 8 weeks of gestation. What is an appropriate management approach for this type of abortion? A. Prepare the woman for a dilation and curettage (D&C). B. Place the woman on bed rest for at least 1 week and reevaluate. C. Prepare the woman for an ultrasound and bloodwork. D. Comfort the woman by telling her that if she loses this baby, she may attempt to get pregnant again in 1 month.

Repetitive transvaginal ultrasounds and measurement of human chorionic gonadotropin (hCG) and progesterone levels may be performed to determine if the fetus is alive and within the uterus. If the pregnancy is lost, the woman should be guided through the grieving process. D&C is not considered until signs of the progress to inevitable abortion are noted or the contents are expelled and incomplete. Bed rest is recommended for 48 hours initially. Telling the client that she can get pregnant again soon is not a therapeutic response because it discounts the importance of this pregnancy. C

What are some safety measures to take while pregnant? Select all that apply. A. Use correct body mechanics. B. Avoid travel to high-altitude regions above 1000 feet. C. Perform activities requiring coordination, balance, and concentration. D. Take rest periods; reschedule daily activities to meet rest and relaxation needs. E. Avoid environmental teratogens, such as cleaning agents, paints, sprays, herbicides, and pesticides. F. Use safety features on tools and vehicles (e.g., safety seat belts, shoulder harnesses, headrests, goggles, helmets) as specified.

Safety measures to take while pregnant include the following: use correct body mechanics; avoid travel to high-altitude regions above 12,000 feet (not 1000 feet); avoid (not perform) activities requiring coordination, balance, and concentration; take rest periods and reschedule daily activities to meet rest and relaxation needs; avoid environmental teratogens, such as cleaning agents, paints, sprays, herbicides, and pesticides; and use safety features on tools and vehicles (e.g., safety seat belts, shoulder harnesses, headrests, goggles, helmets) as specified. A, D, E, F

For a woman at 42 weeks of gestation, which finding requires more assessment by the nurse? A. Fetal heart rate of 116 beats/min B. Cervix dilated 3 cm and 50% effaced C. Score of 8 on the biophysical profile D. One fetal movement noted in 1 hour of assessment by the mother

Self-care in a postterm pregnancy should include performing daily fetal kick counts three times per day. The mother should feel four fetal movements per hour. If she feels fewer than four movements, she should count for 1 more hour. Fewer than four movements in that hour warrants evaluation. The findings described in the other choices are normal at 42 weeks of gestation. D

For a woman at 42 weeks of gestation, which finding requires more assessment by the nurse? A. Fetal heart rate of 116 beats/minute B. Cervix dilated 2 cm and 50% effaced C. Score of 8 on the biophysical profile D. One fetal movement noted in 1 hour of assessment by the mother

Self-care in a postterm pregnancy should include performing daily fetal kick counts three times per day. The mother should feel four fetal movements per hour. If the mother has felt fewer than four movements, she should count for 1 more hour. Fewer than four movements in that hour warrant evaluation. A fetal heart rate of 116 beats/minute is a normal finding at 42 weeks of gestation. Cervical dilation of 2 cm with 50% effacement is a normal finding in a 42-week gestation woman. A score of 8 on the BPP is a normal finding in a 42-week gestation pregnancy. D

The ultrasound report of a 12-week pregnant woman shows snowstorm pattern. Upon further examination, the nurse finds elevated human chorionic gonadotropin (hCG) levels and dark brown vaginal discharge. What complication does the nurse expect in the client? A. Hemorrhage B. Hypertension C. Hyperglycemia D. Molar pregnancy

Snowstorm pattern in the ultrasound, elevated hCG, and dark brown vaginal discharge indicate that the client has a hydatidiform mole. The risk of hemorrhage is predominant in a client with placenta previa. The blood pressure of the client is not affected by the hydatidiform mole. Therefore, the client would not be at a risk of hypertension. a hydatidiform mole does not alter the blood glucose levels. Therefore, the client would not necessarily have hyperglycemia. D

While assessing a postpartum woman, the nurse finds dark red blood coming from the vagina. What can the nurse infer about the client's condition by observing the blood's color? A. The client has partial placental separation. B. The client has a deep laceration of the cervix. C. The client has superficial lacerations of the birth canal. D. The client has disseminated intravascular coagulation (DIC).

Superficial lacerations of the birth canal are characterized by dark red blood oozing from the vagina. The dark red color indicates its venous origin. Partial placental separation is characterized by spurts of blood with clots. Deep laceration of the cervix is characterized by bright redarterial blood. DIC is a condition in which the blood fails to clot or remain clotted. If the client had DIC, the color of the vaginal blood would be bright red. C

A dietician has asked a pregnant client to eat 12 ounces of fish every day. The nurse advises the client to avoid fish such as swordfish, tilefish, and king mackerel. Which fetal complication is the nurse trying to prevent by giving this suggestion? A. Impaired bone development B. Impaired protein metabolism C. Impaired hemoglobin formation D. Impaired neurologic development

Swordfish, tilefish, and king mackerel are known to have high mercury content. Consumption of these fish may lead to an increase in serum levels of mercury, which is neurotoxic to the fetus. Impaired bone development may be caused by inadequate calcium and vitamin D intake, but this is unrelated to the consumption of fish. Inadequate consumption of magnesium and vitamin B6 would result in impaired protein metabolism in the fetus, but this is not directly related to fish consumption. Inadequate intake of iron would result in impaired hemoglobin formation in the fetus. D

Which test is used to determine the presence of fetal-to-maternal bleeding in a pregnant patient? D-dimer test Non-stress test (NST) Kleihauer-Betke (KB) test Biophysical profile (BPP) test

The KB test is used to determine the presence of fetal-to-maternal bleeding or transplacental hemorrhage. The D-dimer test is used to diagnose blood disorders, such as disseminated intravascular coagulation (DIC). The NST and BPP tests are used to determine fetal surveillance. C

Which statement by the nursing student about the management of reduced cervical competence (premature dilation of the cervix) in a pregnant patient indicates effective learning? A. "Progesterone supplementation is the only effective treatment." B. "An abdominal cerclage is performed at the first week of gestation." C. "Surgical treatment is ineffective in patients with extremely short cervix." D. "A prophylactic cerclage is used to constrict the internal os of the cervix."

The best treatment option for premature dilation of the cervix is to surgically place a prophylactic cerclage to constrict the internal os of the cervix. It is usually placed at 11 to 15 weeks of gestation. Progesterone supplementation may not be effective in constricting the cervix and surgical intervention may be necessary. An abdominal cerclage is necessary in case of a failed vaginal cerclage and is usually placed at 11 to 13 weeks of gestation in patients by means of a laparotomy. In patients with an extremely short cervix, an abdominal cerclage is used, which is followed by a cesarean birth. D

A client who has postpartum bleeding due to uterine subinvolution has not recovered from drug therapy. Which procedure would be helpful in managing the bleeding in this client? A. Massaging the abdomen B. Using a nonrebreather mask C. Indwelling a urinary catheter D. Inserting a fist into the vagina

The client is continuously bleeding despite conventional therapy. In this situation, the client is subjected to bimanual compression. In this procedure the fist is inserted into the vagina and knuckles are pressed against the anterior side of the uterus. This maneuver positions the uterus in the pelvic cavity. Uterine massage is also helpful in managing postpartum hemorrhage (PPH). However, uterine massage is not done by massaging the abdomen, but by placing one hand on the abdomen and massaging the posterior side of the uterus. A urinary catheter is indwelled to determine the urinary output. A nonrebreather face mask is given to the client when the client has hypoxemia. D

The nurse talks to a client who maintains a vegan diet about food choices to prevent calcium deficiency. Which foods would the nurse suggest for this purpose? Select all that apply. A. Cheese B. Collards C. Carrots D. Dried figs E. Cooked dried beans

The client is vegan and therefore would not be consuming any dairy products. In order to prevent calcium deficiency in the mother and the fetus, the nurse should suggest that the client eat non-dairy foods that are good sources of calcium. The client should consume about 10 to 12 dried figs, which is equivalent to one cup of milk in terms of calcium content. Collards and cooked dried beans are also good sources of calcium. The client is a vegan, so the nurse should not recommend that the client consume dairy products such as cheese. Carrots are not a good source of calcium. B, D, E

It has been determined after ultrasound that a small piece of the placenta remains in the uterus over an hour after birth, causing the fundus not to be firm and excessive bleeding to continue. Because the client delivered a large infant with a small dose of intravenous pain medication, what action should the nurse take? A. Preparing the client for the removal of the retained placental fragment, including the use of anesthesia. B. Encouraging the consumption of oral fluids to expand the fluid volume. C. Preventing the mother from nursing her infant until her vital signs are stable. D. Encouraging the mother to nurse as much as possible to clamp down the fundal vessels.

The client will need to have the retained placental fragment removed under anesthesia because of the time period since delivery. The client should be nothing by mouth (NPO) at this time because of the expected anesthesia. The infant can nurse, but the retained placental fragment will not allow the uterus to contract. After removal of a retained placenta, the woman is at continued risk for primary pulmonary hypertension (PPH) and infection. A

While performing a physical examination of a postpartum client, the nurse finds that the uterus is firm and contracted. The client reports dark red lochial discharge. Which treatment strategy may help prevent further complications in the client? A. Tocolytics B. Stool softeners C. Anticoagulants D. Halogenated anesthetics

The client's uterus is firm and contracted, which is a normal finding. Slow discharge of dark red blood indicates superficial laceration. Therefore, stool softener would help to establish a normal bowel tone without straining the laceration. Tocolytics are not given to the client, because they dilate the uterine muscles, which may pressurize the laceration. Anticoagulants are given in the case of thromboembolytic defects in the client. Halogenated anesthetics are given to clients to relax the uterine muscles while correcting the uterine inversion B

When a nurse observes profuse postpartum bleeding, the first and most important nursing intervention is to what? A. Call the woman's primary health care provider. B. Administer the standing order for an oxytocic. C. Palpate the uterus and massage it if it is boggy. D. Assess maternal blood pressure and pulse for signs of hypovolemic shock.

The most important nursing intervention is to stop the bleeding. Once the nurse has applied firm massage of the uterine fundus, the primary health care provider should be notified or the nurse can delegate this task to another staff member. This intervention is appropriate after assessment and immediate steps have been taken to control the bleeding. The initial management of excessive postpartum bleeding is firm massage of the uterine fundus. Vital signs will need to be ascertained after fundal massage has been applied. C

Which behavior indicates that a woman is "seeking safe passage" for herself and her infant? A. She keeps all prenatal appointments. B. She "eats for two." C. She drives her car slowly. D. She wears only low-heeled shoes.

The goal of prenatal care is to foster a safe birth for the infant and mother. Keeping all prenatal appointments is a good indication that the woman is indeed seeking "safe passage." Eating properly, driving carefully, using proper body mechanics, and wearing appropriate footwear during pregnancy are healthy measures that all pregnant women should take. A

During a prenatal visit, the nurse finds that the client has decreased mobility and symptoms of preterm labor. Which nursing intervention is to be followed to prevent thrombophlebitis? A. Teach gentle lower extremity exercises to the client. B. Suggest the client to lie in the supine position in bed. C. Provide a calm and soothing atmosphere to the client. D. Give tocolytic medications as per the physician's prescription.

The health care provider may recommend reduced activity for the client experiencing preterm labor, depending on the severity of the symptoms. As a result, the client may be at risk for thrombophlebitis due to limited activity. The nurse should teach the client how to perform gentle exercises of the lower extremities. Suggesting that the client lie in the supine position may cause supine hypotension. Instead, the nurse can suggest that the client lie in a side-lying position to help enhance placental perfusion. The nurse can provide a calm and soothing atmosphere to facilitate coping so as to reduce the client's anxiety, but this intervention does not prevent thrombophlebitis. Tocolytic medications are given to the client to inhibit uterine contractions (UCs), but they do not prevent thrombophlebitis. A

The first and most important nursing intervention when a nurse observes profuse postpartum bleeding is to: A. Call the woman's primary health care provider. B. Administer the standing order for an oxytocic. C. Palpate the uterus and massage it if it is boggy. D. Assess maternal blood pressure and pulse for signs of hypovolemic shock.

The initial management of excessive postpartum bleeding is firm massage of the uterine fundus to stop the bleeding. This is the most important nursing intervention. Then the primary health care provider should be notified or the nurse can delegate this task to another staff member. Administering an oxytocic and ascertaining vital signs are appropriate after assessment has been made and immediate steps have been taken to control the bleeding. C

The nurse is caring for a 32-year-old pregnant client who had an onset of labor at 40 weeks' gestation. Following the labor, the nurse finds that the newborn has a low birth weight (LBW). What explanation will the nurse give to the client as to the etiology of the newborn's LBW? A. Preterm labor. B. Maternal age. C. Diabetic condition of the patient. D. Intrauterine growth restriction (IUGR).

The low birth weight of the newborn is due to IUGR, a condition of inadequate fetal growth. It may be caused due to various conditions, such as gestational hypertension that interferes with uteroplacental perfusion. Interference with uteroplacental perfusion limits the flow of nutrients into the fetus and causes the low birth weight. The onset of labor is at 40 weeks' gestation. Therefore, it is not a preterm labor. The client's age is normal for pregnancy. Therefore, the client's age is not a reason for the low birth weight of the child. Infants born to clients with diabetes would have a high birth weight, not a low one. D

A nurse providing care to a woman in labor should be aware that cesarean birth: A. Is declining in frequency in the United States. B. Is more likely to be performed in the poor in public hospitals who do not receive the nurse counseling that wealthier clients do. C. Is performed primarily for the benefit of the fetus. D. Can be either elected or refused by women as their absolute legal right.

The most common indications for cesarean birth are danger to the fetus related to labor and birth complications. Cesarean births are increasing in the United States. Wealthier women who have health insurance and who give birth in a private hospital are more likely to experience cesarean birth. A woman's right to elect cesarean surgery is in dispute, as is her right to refuse it if in doing so she endangers the fetus. Legal issues are not absolutely clear. C

A nurse providing care to a woman in labor should be aware of which fact about cesarean birth? A. It is declining in frequency in the United States. B. It is more likely to be done for the poor in public hospitals who do not get the nurse counseling that wealthier patients do. C. It is performed primarily for the health of the mother and fetus. D. It can be either elected or refused by women as their absolute legal right.

The most common indications for cesarean birth are to preserve the health of the mother and fetus. Cesarean births are increasing in the United States. Women who have health insurance and who give birth in a private hospital are more likely to experience cesarean birth. A woman's right to elect cesarean birth is in dispute, as is her right to refuse it if in doing so she endangers the fetus. Legal issues are not absolutely clear. C

A pregnant client with chronic hypertension is at risk for placental abruption. Which symptoms of abruption does the nurse instruct the client to be alert for? Select all that apply. A. Weight loss B. Abdominal pain C. Vaginal bleeding D. Shortness of breath E. Uterine tenderness

The nurse instructs the pregnant client to be alert for abdominal pain, vaginal bleeding, and uterine tenderness, because they indicate placental abruption. Weight loss indicates fluid and electrolyte loss, not placental abruption. Shortness of breath indicates inadequate oxygen, which is usually seen in a client who is having cardiac arrest. B, C, E

A pregnant woman at 10 weeks of gestation jogs three or four times per week. She is concerned about the effect of exercise on the fetus. The nurse should tell her: A. "You don't need to modify your exercising any time during your pregnancy." B. "Stop exercising, because it will harm the fetus." C. "You may find that you need to modify your exercise to walking later in your pregnancy, around the seventh month." D. "Jogging is too hard on your joints; switch to walking now."

The nurse should inform the woman that she may need to reduce her exercise level as the pregnancy progresses. Typically, running should be replaced with walking around the seventh month of pregnancy. Physical activity promotes a feeling of well-being in pregnant women. It improves circulation, promotes relaxation and rest, and counteracts boredom. Simple measures should be initiated to prevent injuries, such as warm-up and stretching exercises, to prepare the joints for more strenuous exercise. C

The nurse is caring for a client with premature rupture of membranes (PROM). How should the nurse instruct the client to manage the situation? A. "Consume excess amounts of fluids." B. "Assess fetal movement on a daily basis." C. "Monitor the skin for any discoloration." D. "Place yourself in Trendelenburg position."

The nurse should instruct a pregnant client with PROM to perform daily fetal movement counts. Reduction in fetal movements indicates fetal dysfunction. Clients who are administered tocolytic agents, such as nifedipine (Adalat), are instructed to consume excess fluids to prevent effects of vasodilatation. Consumption of excess fluids is unrelated to the management of PROM. Skin discoloration is observed in conditions like jaundice, but not in clients with PROM. The nurse places the client in Trendelenburg position if the client has symptoms of umbilical cord prolapse. B

The nurse finds that despite gentle traction to the umbilical cord and uterine massage, the client's placenta has not expelled out even after 30 minutes of childbirth. The primary health care provider instructs the nurse to administer nitroglycerin IV (Nitrostat) to the client. What could be the reason for this instruction? Prevention pelvic hematoma To increase the effects of regional anesthesia To promote uterine relaxation Prevention of postpartum hemorrhage

The placenta is usually expelled out within 30 minutes after birth. Retained placenta is a condition in which the placenta is not expelled within 30 minutes after birth despite using manual measures, such as gentle traction on the umbilical cord and uterine massage. In this condition nitroglycerin IV (Nitrostat) is administered to the patient to promote uterine relaxation. Nitroglycerin IV (Nitrostat) does not affect blood coagulation; therefore, it does not prevent pelvic hematoma. Nitroglycerin IV (Nitrostat) is not an anesthetic agent; therefore, it does not provide regional anesthesia. Nitroglycerin IV (Nitrostat) does not cause uterine contractions (UCs), so it does not prevent PPH. C

The nurse is reviewing the lab reports of a client who is 10 weeks pregnant and has a family history of diabetes mellitus. The nurse finds that the client's 1-hour glucose tolerance test is normal. What does the nurse advise the client? A. Undergo a renal function test B. Increase food intake C. Repeat the test at 28 weeks D. Undergo a 3-hour glucose test

The pregnant client has a family history of diabetes and may be at a high risk of developing gestational diabetes. Because the initial 1-hour glucose tolerance test results are normal, the client should be advised to repeat the test again at 28 weeks of pregnancy. The client has normal blood sugar levels and is therefore unlikely to have renal complications. The client does not need to undergo a renal function test. The lab reports do not indicate that the client has any nutritional deficiencies and does not indicate a need for the client to increase her food intake. A 3-hour glucose test is conducted only for pregnant clients whose 1-hour glucose tolerance test is positive. C

A pregnant client in the third trimester reports leg cramps. Which interventions may not need a primary health care provider's instructions? Select all that apply. A. Advise the client to dorsiflex her foot. B. Advise the client to stand on a cold surface. C. Advise the client to take calcium lactate tablets. D. Advise the client to take aluminum hydroxide gel with meals. E. Advise the client to use massage and heat over the affected muscles.

The pregnant client may develop leg cramps during the third trimester because of the growing uterus and a reduced level of serum calcium or an elevation of serum phosphorus. Dorsiflexion of the foot decreases the angle between the dorsum of the foot and the leg and relaxes calf spasms. Standing on a cold surface helps relieve pain. Massage and heat over the affected muscle provides relief to the client. Calcium lactate tablets and aluminum hydroxide gel should be used only after consulting the primary health care provider, because these drugs have the potential to cause complications. These medications help relieve leg cramps by reducing phosphorus levels. A, B, E

In planning for an expected cesarean birth for a woman who has given birth by cesarean section previously and who has a fetus in the transverse presentation, the nurse includes which information? A. "Because this is a repeat procedure, you are at the lowest risk for complications." B. "Even though this is your second cesarean birth, you may wish to review the preoperative and postoperative procedures." C. "Because this is your second cesarean birth, you will recover faster." D. "You will not need preoperative teaching because this is your second cesarean birth."

The statement in B is most appropriate. The statements in A, C, and D are not accurate. Maternal and fetal risks are associated with every cesarean section. Physiologic and psychologic recovery from a cesarean section is multifactorial and individual to each client each time. Preoperative teaching should always be performed regardless of whether the client has already had this procedure. B

The nurse is caring for a pregnant client in her first trimester with imbalanced nutrition due to nausea and vomiting. What nursing interventions will help maintain appropriate nutrition in the client? Select all that apply. A. Advise the client to rest as needed. B. Advise the client to increase fiber in her diet. C. Advise the client to consume small and frequent meals. D. Advise the client to eat dry crackers first thing in the morning. E. Advise the client to contact the primary health care provider if vomiting is severe.

The pregnant client may eat less than her body requires in the first trimester due to nausea and vomiting. The nurse should advise the client to eat small and frequent meals to avoid nausea. Eating dry crackers first thing in the morning will help decrease the incidence of vomiting. The nurse should advise the client to contact the primary health care provider if severe vomiting occurs. The primary health care provider can help identify possible causes of hyperemesis. Resting reduces fatigue in the client. Encouraging the client to increase fiber in her diet is an intervention to avoid constipation. C, D, E

*The nurse is assessing a pregnant client who is in the second trimester. The client tells the nurse, "My body shook for a while when I was sitting on my couch." What laboratory parameter would the nurse monitor?* A. Blood glucose levels B. Blood pressure C. Complete blood cell count D. Electroencephalogram (EEG)

The pregnant client may experience mild convulsions caused by elevated blood pressure. Therefore, the nurse should monitor the client's blood pressure. Fluctuations in blood glucose levels may cause hyperglycemia or hypoglycemia during pregnancy; these fluctuations do not cause seizures. The complete blood cell count is used to determine the presence of infection and anemia. These conditions are not known to cause seizures. EEG is used to determine brain functioning in clients with chronic seizures. It is not necessary in this case. The assumption here is that "shook" qualifies as a seizure but this is unclear. A patient can have high or low blood sugar and this could lead to "tremors" and/or "shaking" type behavior. B

The nurse is caring for a pregnant client who reports fatigue due to anemia. Which nursing intervention helps manage anemia in the client? A. Advise the client to rest as needed. B. Advise the client to eat a well-balanced diet. C. Discuss the use of support systems with the client. D. Reassure the client of the transitory nature of fatigue.

The pregnant client may feel fatigued during the first trimester due to hormonal changes. The nurse should advise the client to eat a well-balanced diet to meet the increased metabolic demands and avoid anemia. Advising the client to rest and use support systems, and emphasizing the transitory nature of fatigue, reduces fatigue in the client but has no effect in treating anemia. B

The nurse is assessing a postpartum client 4 hours after delivery. The nurse observes that the client has cool, pale, and clammy skin with severe restlessness and thirst. What should the immediate nursing intervention be? A. Begin fundal massage and start oxygen therapy. B. Begin an hourly pad count and reassure the client. C. Elevate the head of the bed and assess vital signs. D. Assess for hypovolemia and notify the primary health care provider.

The presence of cool, pale, and clammy skin with severe restlessness and thirst are the diagnostic signs of hypovolemia. If the nurse observes these signs it should be immediately reported to the primary health care provider. Fundal massage will only be effective for clients with postpartum hemorrhage due to uterine bleeding. Pad count is associated with postpartum hemorrhage and is unrelated to cold and clammy skin. Elevating the head of the bed may affect the client's vital signs. However, it is not associated with hypovolemia. D

Which of these are presumptive indicators of pregnancy? Select all that apply. A. Dysmenorrhea B. Morning sickness C. Breast tenderness D. Urinary incontinence E. Breast and abdominal enlargement F. Skin changes, such as striae gravidarum, deeper pigmentation of the areola, melasma (mask of pregnancy), and linea nigra (pigmented line on the abdomen)

The presumptive indicators of pregnancy include: morning sickness; breast tenderness; breast and abdominal enlargement; and skin changes, such as striae gravidarum, deeper pigmentation of the areola, melasma (mask of pregnancy), and linea nigra (pigmented line on the abdomen). Amenorrhea (not dysmenorrhea) and urinary frequency (not urinary incontinence) are also presumptive indicators of pregnancy. B, C, E, F

Which factors would lead to an increased likelihood of uterine rupture? (Select all that apply.) A. Preterm singleton pregnancy B. G3P3 with all vaginal deliveries C. Short interval between pregnancies D. Patient receiving a trial of labor (TOL) following a VBAC delivery E. Patient who had a primary cesarean section with a classic incision

The shorter the interval between pregnancies/deliveries, the higher the risk of uterine rupture. A patient who is having a TOL following a VBAC and a patient who has had a C section with a classic incision into the uterus are at increased risk for uterine rupture. A pregnant woman with a singleton pregnancy (one fetus), even if preterm, is not considered to be at increased risk for uterine rupture; nor is a multipara who has delivered all her infants vaginally. C, D, E

What are the signs and symptoms of supine hypotension? Select all that apply. A. Pallor B. Acid reflux C. Dizziness D. Tachycardia E. Breathlessness F. Dry, warm skin

The signs and symptoms of supine hypotension include pallor, nausea (not acid reflux), dizziness, tachycardia, breathlessness, and cold, clammy, sweaty skin (not dry, warm skin). A, C, D, E

What are the signs and symptoms of supine hypotension? Select all that apply. A. Pallor B. Acid reflux C. Dizziness D. Tachycardia E. Breathlessness

The signs and symptoms of supine hypotension include pallor, nausea (not acid reflux), dizziness, tachycardia, breathlessness, and cold, clammy, sweaty skin (not dry, warm skin). A, C, D, E

With regard to medications, herbs, shots, and other substances normally encountered, the maternity nurse should be aware that during pregnancy: A. Prescription and over-the-counter (OTC) drugs that otherwise are harmless can be made hazardous by metabolic deficiencies of the fetus. B. The greatest danger of drug-caused developmental deficits in the fetus is seen in the final trimester. C. Killed-virus vaccines (e.g., tetanus) should not be given, but live-virus vaccines (e.g., measles) are permissible. D. No convincing evidence exists that secondhand smoke is potentially dangerous to the fetus.

The statement in A is especially true for new medications and combinations of drugs. The greatest danger of drug-caused developmental defects exists in the interval from fertilization through the first trimester, when a woman may not realize that she is pregnant. Live-virus vaccines should be part of postpartum care; killed-virus vaccines may be administered during pregnancy. Secondhand smoke is associated with fetal growth restriction and increases in infant mortality. A

An expectant father confides in the nurse that his pregnant wife, at 10 weeks of gestation, is driving him crazy. "One minute she seems happy, and the next minute she is crying over nothing at all. Is there something wrong with her?" The nurse's best response is: A. "This is normal behavior and should begin to subside by the second trimester." B. "She may be having difficulty adjusting to pregnancy; I will refer her to a counselor I know." C. "This is called emotional lability and is related to hormone changes and anxiety during pregnancy. The mood swings will eventually subside as she adjusts to being pregnant." D. "You seem impatient with her. Perhaps this is precipitating her behavior."

The statement in C is the most appropriate response because it gives an explanation and a time frame for when the mood swings may stop. The statement in A is an appropriate response but it does not answer the father's question. Mood swings are a normal finding in the first trimester; the woman does not need counseling. The statement in D is judgmental and not appropriate. C

After giving birth to a stillborn infant, the woman turns to the nurse and says, "I just finished painting the baby's room. Do you think that caused my baby to die?" The nurse's best response to this woman is: A. "That's an old wives' tale; lots of women are around paint during pregnancy, and this doesn't happen to them." B. "That's not likely. Paint is associated with elevated pediatric lead levels." C. Silence. D. "I can understand your need to find an answer to what caused this. What else are you thinking about?"

The statement in D is very appropriate for the nurse. It demonstrates caring and compassion and allows the mother to vent her thoughts and feelings, which is therapeutic in the process of grieving. The nurse should resist the temptation to give advice or to use clichés in offering support to the bereaved. Trying to give bereaved parents answers when no clear answers exist or trying to squelch their guilt feelings does not help the process of grief. Additionally the response in B probably would increase the mother's feelings of guilt. One of the most important goals of the nurse is to validate the experience and feelings of the parents by encouraging them to tell their stories, and listening with care, which silence would not do. D

A pregnant woman's amniotic membranes rupture. Prolapsed cord is suspected. Which intervention is the nurse's top priority? A. Place the woman in the knee-chest position. B. Cover the cord in a sterile towel saturated with warm normal saline. C. Prepare the woman for a cesarean birth. D. Start oxygen by face mask.

The woman is assisted into a position (e.g., modified Sims position, Trendelenburg position, or knee-chest position) in which gravity keeps the pressure of the presenting part off the cord. Relieving pressure on the cord is the nursing priority. The nurse may also use her gloved hand or two fingers to lift the presenting part off the cord. If the cord is protruding from the vagina it may be covered with a sterile towel soaked in saline. The nurse should administer O2 by facial mask at 8 to 10 L/min until delivery is complete. If the cervix is fully dilated, the nurse should prepare for immediate vaginal delivery. Cesarean birth is indicated only if cervical dilation is not complete. A

A 22-year-old woman pregnant with a single fetus has a preconception body mass index (BMI) of 24. When she was seen in the clinic at 14 weeks of gestation, she had gained 1.8 kg (4 lbs) since conception. How should the nurse interpret this? A. This weight gain indicates possible gestational hypertension. B. This weight gain indicates that the woman's infant is at risk for intrauterine growth restriction (IUGR). C. This weight gain cannot be evaluated until the woman has been observed for several more weeks. D. The woman's weight gain is appropriate for this stage of pregnancy.

This woman's BMI is within the normal range. During the first trimester, the average total weight gain is only 1 to 2.5 kg. The desirable weight gain during pregnancy varies among women. The primary factor to consider in making a weight gain recommendation is the appropriateness of the prepregnancy weight for the woman's height. A commonly used method of evaluating the appropriateness of weight for height is the BMI. This woman has gained the appropriate amount of weight for her size at this point in her pregnancy. Weight gain should take place throughout the pregnancy. The optimal rate of weight gain depends on the stage of the pregnancy. D

Which statement about multifetal pregnancy is not accurate? A. The expectant mother often experiences anemia because the fetuses have a greater demand for iron. B. Twin pregnancies come to term with the same frequency as single pregnancies. C. The mother should be counseled to increase her nutritional intake and gain more weight. D. Backache and varicose veins are often more pronounced.

Twin pregnancies often end in prematurity; serious efforts should be made to bring the pregnancy to term. A woman with a multifetal pregnancy often experiences anemia because of the increased demands of two fetuses; this issue should be monitored closely throughout her pregnancy. The client may need nutrition counseling to ensure that she gains more weight than what is needed for a singleton birth. The considerable uterine distention in multifetal pregnancy is likely to cause backache and leg varicosities; maternal support hose should be recommended. B

In which situations would the use of Methergine or prostaglandin be contraindicated even if the patient was experiencing a postpartum significant bleed? (Select all that apply.) A. Patient has delivered twin pregnancies. B. Patient's blood pressure postpartum is 180/90. C. Patient has a history of asthma. D. Patient has a mitral valve prolapse. E. Patient is a grand multip.

Twin pregnancies successfully delivered and grand multiparity are not contraindications to the use of these medications. If a patient is hypertensive or has cardiovascular disease, these medications would not be used. If a patient has a history of asthma, prostaglandin medication would not be used. B, C, D

Which suggestions should the nurse include when teaching about appropriate weight gain in pregnancy? Select all that apply. A. Underweight women should gain 12.7 to 18.1 kg (28-40 lb). B. Obese women should gain at least 6.8 to 11.3 kg (15-25 lb). C. Adolescents are encouraged to strive for weight gains at the upper end of the recommended scale. D. In twin gestations, the weight gain recommended for a single fetus pregnancy should simply be doubled. E. Normal weight women should gain 11.3 to 15.9 kg (25-35 lb).

Underweight women need to gain the most. Obese women need to gain weight during pregnancy to equal the weight of the products of conception. Adolescents are still growing; therefore, their bodies naturally compete for nutrients with the fetus. Women bearing twins need to gain more weight (usually 21 to 28 kg [46 to 62 lb]) but not necessarily twice as much. Normal weight women should gain 11.3 to 15.9 kg (25-35 lb). A, B, C, E

An expectant couple asks the nurse about intercourse during pregnancy and whether it is safe for the baby. The nurse should tell the couple that: A. Intercourse should be avoided if any spotting from the vagina occurs afterward. B. Intercourse is safe until the third trimester. C. Safer-sex practices should be used once the membranes rupture. D. Intercourse and orgasm are often contraindicated if a history or signs of preterm labor are present.

Uterine contractions that accompany orgasm can stimulate labor and would be problematic if the woman were at risk for or had a history of preterm labor. Some spotting can normally occur as a result of the increased fragility and vascularity of the cervix and vagina during pregnancy. Intercourse can continue as long as the pregnancy is progressing normally. Safer-sex practices are always recommended; rupture of the membranes may require abstaining from intercourse. D

A woman at 39 weeks of gestation with a history of preeclampsia is admitted to the labor and birth unit. She suddenly experiences increased contraction frequency of every 1 to 2 minutes; dark red vaginal bleeding; and a tense, painful abdomen. The nurse suspects the onset of what condition? A. Eclamptic seizure B. Rupture of the uterus C. Placenta previa D. Placental abruption

Uterine tenderness in the presence of increasing tone may be the earliest finding of premature separation of the placenta (abruptio placentae or placental abruption). Women with hypertension are at increased risk for an abruption. Eclamptic seizures are evidenced by the presence of generalized tonic-clonic convulsions. Uterine rupture presents as hypotonic uterine activity, signs of hypovolemia, and in many cases the absence of pain. Placenta previa presents with bright red, painless vaginal bleeding. D

The nurse examines a client at 30 weeks of gestation for cervical dilation. The nurse understands that the infant may be at risk of cerebral palsy if it is born preterm. Which intervention would help to prevent cerebral palsy? A. Shifting the client to an obstetric facility B. Administering antibiotic medications to the client C. Administering antenatal glucocorticoids to the client D. Administering magnesium sulfate (Epsom salts) to the client

When preterm birth appears inevitable, magnesium sulfate (Epsom salts) is administered to the client at 24 to 32 weeks of gestation to prevent the risk of cerebral palsy. Clients in preterm labor should be shifted to a healthcare facility that is well-equipped to handle emergencies and take care of preterm infants. Antibiotics are administered to prevent infections. Antenatal glucocorticoids are administered to pregnant clients to prevent the risk of respiratory depression in the fetus, caused by structurally and functionally immature lungs. D

What question does the nurse ask when assessing the socioeconomic status of a pregnant client? "What prescription medications do you take?" "Do you have any factories around your house?" "Do you have any medical insurance?" "Are there any diseases that run in your family?"

When the nurse is assessing a client's socioeconomic status, the nurse should determine if the client has health insurance. Lack of health insurance may mean the client does not have a job to pay for insurance nor the income to pay for it privately. This may impact the client's prenatal care if she cannot afford services. When the nurse asks about the family's medical history, this falls under the client's personal history. The nurse asks about the community in which the patient lives when assessing the client's environment. Medications can affect the fetus in a pregnant client. Therefore, the nurse should ask about the medications taken by the client when assessing the patient's health status. C

A pregnant client is diagnosed with succenturiate placenta. What should the nurse understand about the client's condition? A. The placenta separates prematurely from the uterus. B. The placenta is implanted in the lower uterine segment. C. The placental trophoblasts have undergone proliferative growth. D. The placenta is divided into more lobes, rather than a single lobe.

When the placenta is divided into two or more lobes, rather than as a single mass, it is called a succenturiate placenta. Premature separation of the placenta from the uterus is called abruptio placentae. A placenta implanted in the lower uterine segment is known as placenta previa. Benign proliferative growth of the placental trophoblast is known as molar pregnancy. D

*The nurse is assisting the primary health care provider during a pelvic examination of a pregnant client. What does the nurse assess while performing a pelvic examination?* Size of the uterus Height of the fundus Client's knowledge of Kegel exercises Tone of pelvic musculature

Whenever a pelvic examination is being performed for a pregnant client, it is important to assess her knowledge of Kegel exercises, which help maintain the tone of pelvic musculature. The size of the uterus, the height of the fundus, and the tone of pelvic musculature are assessed by the primary health care provider during a pelvic examination.

What measures will the nurse implement in her postpartum client to prevent postpartum infection? Select all that apply. A. Instruct the client to consume foods that are rich in iron. B. Instruct the client to wash hands with soap after urination. C. Perform fundal massage on the client when necessary. D. Instruct the client to change the perineal pads from front to back. E. Inform the client to avoid consuming an excess amount of water.

While teaching a client about preventive measures for postpartum infection, the nurse should emphasize diet pattern and hygiene. An iron-rich diet is recommended to prevent anemia during hemorrhage. Washing one's hands with soap will prevent the spread of infection. Fundal massage is performed during the postpartum period to promote uterine contraction.The perineal pads should be changed from front to back, because this prevents postpartum urinary tract infections by preventing the deposit of the microorganisms present in the lochia near the urethra. The postpartum client is advised to drink plenty of water to maintain hydration and prevent infection in the uterus. A, B, C, D

Which suggestion about weight gain is not an accurate recommendation? A. Underweight women should gain 12.5 to 18 kg. B. Obese women should gain at least 7 kg. C. Adolescents are encouraged to strive for weight gains at the upper end of the recommended scale. D. In twin gestations, the weight gain recommended for a single fetus pregnancy should simply be doubled.

Women bearing twins need to gain more weight (usually 16 to 20 kg) but not necessarily twice as much. Underweight women need to gain the most. Obese women need to gain weight during pregnancy to equal the weight of the products of conception. Adolescents are still growing; therefore, their bodies naturally compete for nutrients with the fetus. D


संबंधित स्टडी सेट्स

National Electric Code (NEC) Article 250.

View Set

Quiz 1 Financial Management (Ch 1 and Ch 4)

View Set

BUAD 280: Closing Entries, Chapters 5-6 and 8-9

View Set

MEDR 4514 — Module 8 basics?? i suppose?

View Set

(G) Give the meaning of the following PREFIXES

View Set